EPPP - Practice Exam 2

  1. In its early stages, Alzheimer's disease shares several characteristics with Korsakoff's syndrome. Specifically, both disorders are characterized by:

    A.anterograde amnesia that affects declarative memories but not procedural memories.

    B.anterograde amnesia that affects both declarative and procedural memories.

    C.retrograde amnesia for recent and remote declarative (but not procedural) memories.

    D.retrograde amnesia for recent (but not remote) declarative and procedural memories.
    A

    Alzheimer's disease and Korsakoff's syndrome share several characteristics. In Alzheimer's disease, anterograde amnesia for declarative knowledge is often the first neurological finding. It is also characteristic of Korsakoff's syndrome.

    Answer B: Procedural memory is left relatively intact in both disorders, and confabulation is characteristic of Korsakoff's syndrome but not Alzheimer's disease.

    Answer C: In Alzheimer's disease, there is a "flat" retrograde amnesia that emerges once the disorder has progressed to moderate levels of impairment (i.e., the amnesia affects both remote and recent memories). In Korsakoff's syndrome, retrograde amnesia affects recent memories, while remote memories remain intact.
  2. Adults with ADHD tend to have problems related to social relationships and, compared to peers without ADHD:

    A.they have poorer health outcomes but higher educational and occupational achievement.

    B.they have better health outcomes but lower educational and occupational achievement.

    C.they have similar health outcomes and similar educational and occupational achievement.

    D.they have poorer health outcomes and lower educational and occupational achievement.
    The correct answer is D.

    Compared to peers without ADHD, adults who are diagnosed with ADHD tend to have poorer health outcomes and lower levels of educational and occupational achievement. Furthermore, adults with symptoms of ADHD may be diagnosed with comorbid mental health disorders, including anxiety, Bipolar Disorder, depression, antisocial behavior, and substance abuse.

    Answer A: Adults diagnosed with ADHD tend to have lower levels of educational achievement compared to their peers without a diagnosis of ADHD. 

    Answer B: Adults diagnosed with ADHD tend to have poorer health outcomes compared to their peers without a diagnosis of ADHD. 

    Answer C: Compared to peers without ADHD, adults who are diagnosed with ADHD tend to have poorer health outcomes and lower levels of educational and occupational achievement.
  3. Research investigating the efficacy of phototherapy as a treatment for Seasonal Affective Disorder has generally found that it is:

    A.substantially less effective than antidepressant drugs.

    B.an effective treatment and produces results comparable to those produced by antidepressant drugs.

    C.effective only when administered in conjunction with a low- to moderate-dose of an antidepressant drug.

    D.effective only for individuals who have atypical symptoms.
    The correct answer is B.

    Studies have demonstrated that phototherapy (light therapy) is as effective as antidepressants in treating symptoms of Seasonal Affective Disorder (SAD).

    Answer A: The studies indicate that phototherapy is as effective as antidepressants for reducing the symptoms of SAD.

    Answer C: Research results have indicated that phototherapy, on its own, is as effective as antidepressants for reducing symptoms of SAD.

    Answer D: According to research, light therapy is an effective treatment for reducing symptoms of SAD.
  4. The presence of which of the following symptoms would be more suggestive of a diagnosis of Caffeine-Induced Anxiety Disorder than Caffeine Intoxication?

    A.Tachycardia or cardiac arrhythmia

    B.Psychomotor agitation

    C.Panic attacks

    D.Rambling flow of thoughts and speech
    The correct answer is C.

    The differential diagnosis of Caffeine-Induced Anxiety Disorder and Caffeine Intoxication can be difficult due to the overlap between the symptoms of the two disorders. Caffeine-Induced Anxiety Disorder would be the more likely diagnosis when the patient's symptoms of anxiety are in excess of those associated with Caffeine Intoxication - e.g. when the patient has signs of Generalized Anxiety Disorder, panic attacks, or obsessive-compulsive symptoms.

    Answer A: A diagnosis of Caffeine Intoxication requires the presence of at least five of twelve possible symptoms that develop during or shortly after the consumption of caffeine. Tachycardia or cardiac arrhythmia is one of the symptoms of Caffeine Intoxication.

    Answer B: Psychomotor agitation is a symptom of Caffeine Intoxication.

    Answer D: Rambling flow of thoughts and speech is also indicative of Caffeine Intoxication.
  5. Recent research indicates that higher rates of Schizophrenia among African Americans are most likely attributable to:

    A.heredity.

    B.family dynamics.

    C.socioeconomic factors.

    D.misdiagnosis of hallucinations and delusions.
    The correct answer is D.

    Research results indicate that the discrepancy in rates of Schizophrenia is due to misdiagnosis rather than actual differences in the prevalence of the disorder. A contributing factor to misdiagnosis may be that African Americans have a higher incidence of hallucinations and delusions that are due to other disorders such as mania, depression, and alcohol abuse.
  6. The use of Haloperidol or other dopamine blocking agent for the treatment of Tourette's Disorder:

    A.is uncommon since these drugs are rarely effective for alleviating tics.

    B.can be problematic in many cases because of the severe side effects of these drugs.

    C.is contraindicated because these drugs exacerbate tics in most patients.

    D.should be considered only in the presence of psychotic symptoms.
    The correct answer is B.

    Antipsychotic drugs are associated with several unpleasant side effects. About 50% of people with Tourette's Disorder who take Haloperidol or a similar drug develop intolerable side effects. Therefore, Clonidine (an antihypertensive drug) is often prescribed as an alternative because it produces fewer and less severe side effects.

    Answer A: Antipsychotic drugs are effective in about 80% of cases of Tourette's Disorder.

    Answer C: This is not true. Antipsychotic drugs are useful for alleviating tics in most cases.

    Answer D: This is not true.
  7. Intense narcoleptic "sleep attacks" are often accompanied by:

    A.cataplexy.

    B.dyskinesia.

    C.ataxia.

    D.muscular rigidity.
    The correct answer is A.

    Narcolepsy involves frequent intense periods of irresistible sleep. Narcolepsy sleep attacks may include cataplexy, which is a sudden loss of muscle tone.
  8. Researchers have linked depression to:

    A.decreased REM latency and increased REM density.

    B.decreased REM latency and decreased REM density.

    C.increased REM latency and increased REM density.

    D.increased REM latency and decreased REM density.
    The correct answer is A.

    Studies have demonstrated correlations between depression and several alterations in sleep quantity and quality. The research has consistently shown that depression is associated with decreased sleep continuity, reduced slow-wave sleep (stages 3 and 4), shortened REM latency (earlier onset of REM sleep), and increased REM density (increased frequency of rapid eye movements).
  9. The diagnosis of Cyclothymic Disorder requires a minimum duration of symptoms for:

    A.one year in adults and eight months in children and adolescents.

    B.one year in adolescents and adults and six months in children.

    C.two years in adults and one year in children and adolescents.

    D.two years in adolescents and adults and one year in children.
    The correct answer is C.

    You are likely to encounter one or two questions on the licensing exam that require you to know the duration requirements for a specific diagnosis. To prepare for these questions, focus on questions in the AATBS practice tests that address this issue. The minimum duration of symptoms for Cyclothymic Disorder is two years for adults and one year for children and adolescents.
  10. A 58-year-old woman presents to your mental health clinic with her husband. She seems agitated and during your interview with her, she paces back and forth and engages in frequent handwringing. The woman says she's been feeling very sad and hopeless, and she also tells you that she thinks her next-door neighbor has tried to poison her food several times because she has been so unfriendly toward him, although the husband confirms this is untrue. The woman's husband says that his wife has been gradually losing interest in her usual activities over the past several weeks but that her symptoms became worse a few days ago. Based on this information, the most likely diagnosis is:

    A.Major Depressive Disorder.

    B.Schizoaffective Disorder.

    C.Bipolar II Disorder.

    D.Delusional Disorder.
    The correct answer is A.

    The woman's pervasive depressed mood and associated symptoms (agitation and impaired memory) suggest a major depressive episode. Delusions are not uncommon in Major Depressive Disorder and often involve a belief that one is being persecuted because of immorality or personal inadequacy.

    Answer B: In Schizoaffective Disorder, there are periods of at least two weeks in which there have been delusions or hallucinations in the absence of a mood disturbance.

    Answer C: Bipolar II Disorder requires at least one hypomanic episode and a major depressive episode. The woman's symptoms are not suggestive of hypomania.

    Answer D: A diagnosis of Delusional Disorder requires the presence of delusions that are not due to Major Depressive Disorder or another disorder. This woman's delusion seems to be part of her depression.
  11. The presence of which of the following would suggest that the appropriate diagnosis is Social Anxiety Disorder rather than Agoraphobia?

    A.Fear of humiliation or embarrassment in social situations

    B.Increased anxiety in feared situations when accompanied by a trusted companion

    C.Anxiety is perceived as excessive and temporarily controllable

    D.An absence of the physiological symptoms associated with a panic attack
    The correct answer is B.

    Social Anxiety Disorder (also known as Social Phobia) and Agoraphobia are sometimes difficult to distinguish because they share several symptoms. The effects of having a companion in anxiety-arousing situations can help distinguish the two disorders. In Agoraphobia, the presence of a trusted companion often alleviates anxiety. In Social Anxiety Disorder, a companion can actually increase anxiety.

    Answer A: Fear of humiliation or embarrassment in social situations is a feature of both disorders (although the fear is not necessarily limited to social situations in Agoraphobia).

    Answer C: This symptom is not useful for distinguishing between Social Anxiety Disorder and Agoraphobia.

    Answer D: Both disorders may or may not involve panic attacks.
  12. Two weeks after witnessing the murder of a co-worker, Mrs. Cee develops many symptoms, including a sense of detachment, flashbacks of the event, sleep problems, impaired concentration, a depressed mood, and an unwillingness to talk about the murder. She has had these symptoms for seven days. Based on these symptoms, the most likely diagnosis is:

    A.PTSD.

    B.Brief Psychotic Disorder.

    C.Acute Stress Disorder.

    D.Conversion Disorder.
    The correct answer is C.

    A diagnosis of Acute Stress Disorder requires exposure to actual or threatened death, severe injury, or sexual violation. Mrs. Cee's symptoms began after witnessing her co-worker's death. Her symptoms fit into categories of intrusion, negative mood, dissociative, avoidance, and arousal. Moreover, her symptom duration (seven days) falls within three days to one month.

    Answer A: The duration of the woman's symptoms is less than one month (the minimum duration required for PTSD); therefore, Acute Stress Disorder is the most likely diagnosis.

    Answer B: Mrs. Cee is not demonstrating symptoms of psychosis.

    Answer D: Mrs. Cee's symptoms are not somatic.
  13. Interventions for individuals diagnosed with Conduct Disorder are most effective when:

    A.they are combined with CBT or DBT.

    B.they begin during adolescence and include CBT.

    C.they begin before adolescence and include family interventions.

    D.they are combined with psychotropic medications.
    The correct answer is C.

    Interventions for youth diagnosed with Conduct Disorder are most effective when they target preadolescents and when they include family intervention, such as Parent Management Training (PMT), Multisystemic Treatment (MST), or Functional Family Therapy (FFT).
  14. A new client tells the clinician that she has been feeling "down in the dumps" and "completely worthless" for the last five or six days. She also says she is having trouble sleeping, has lost her appetite, and feels anxious "all the time." The client denies having previous mood problems and states that there are no specific stressors in her life. The client's symptoms are most suggestive of which of the following DSM-5 diagnoses?

    A.Major Depressive Disorder

    B.Other Specified Depressive Disorder

    C.Other Specified Anxiety Disorder

    D.Major Depressive Episode, single episode
    The correct answer is B.

    This client has symptoms of Major Depressive Disorder, but they have persisted for less than two weeks, which is the minimum duration required for the diagnosis. Therefore, a diagnosis of Other Specified Depressive Disorder with the reason "short-duration depressive episode" is the best fit for her symptoms.

    Answer A: A diagnosis of Major Depressive Disorder requires a duration of symptoms of at least two weeks.

    Answer C: Although the client is experiencing anxiety, her symptoms do not meet the criteria for an anxiety disorder. Furthermore, she is exhibiting several symptoms that are typical of depression.

    Answer D: Major depressive episode is not a DSM-5 diagnosis but, instead, is an essential feature of Major Depressive Disorder and Bipolar II Disorder, and a possible feature of Bipolar I Disorder.
  15. Which of the following is NOT included as an Alcohol-Related Disorder in the DSM-5?

    A.Alcohol Dependence

    B.Alcohol Intoxication

    C.Alcohol Withdrawal

    D.Alcohol-Induced Major Neurocognitive Disorder
    The correct answer is A.

    The DSM-5 distinguishes Alcohol Use Disorders on a continuum (mild, moderate, severe) based upon the number of applicable criteria. Alcohol Use Disorders replaced DSM-IV diagnoses of Alcohol Dependence and Alcohol Abuse.

    Answer B: Alcohol Intoxication is included as a diagnosis in the DSM-5 and involves maladaptive behavioral and psychological changes with at least one of the following symptoms: slurred speech; unsteady gait; nystagmus; impaired attention or memory.

    Answer C: The DSM-5 describes Alcohol Withdrawal as involving the development of at least two symptoms within several hours to a few days following cessation or reduction of alcohol consumption (e.g., autonomic hyperactivity, hand tremor, insomnia, transient illusions or hallucinations, seizures).

    Answer D: In the DSM-5, Alcohol-Induced Major Neurocognitive Disorder involves a significant decline in one or more cognitive domains that interferes with independence in everyday activities.
  16. Which of the following is not a DSM-5 diagnostic criterion for Conversion Disorder?

    A.One or more symptoms that involve an alteration in sensory or motor functioning

    B.Evidence of incompatibility between the symptoms and known medical conditions

    C.Symptoms are not better explained by a medical condition or other mental disorder

    D.Evidence that the symptoms are related to a psychological need or conflict.
    The correct answer is D.

    This was a requirement for a diagnosis of Conversion Disorders in previous versions of the DSM; however, it is not a diagnostic criterion in DSM-5.

    Answer A: This is a diagnostic criterion for Conversion Disorder.

    Answer B: This DSM-5 includes this criterion for a diagnosis of Conversion Disorder.

    Answer C: This criterion must be met for a diagnosis of Conversion Disorder.
  17. A client tells a clinician that she is having "a lot of problems" with food. She states that she has been on numerous diets over the years and, even when she achieves her desired weight, she always gains it back. She says that she has been evaluated for medical problems such as hypothyroidism, but all the tests have been negative. When determining if the client's symptoms meet the criteria for Binge Eating Disorder or Bulimia Nervosa, the clinician should keep in mind that:

    A.binges are more severe in Binge Eating Disorder than in Bulimia Nervosa.

    B.a disturbance in body image is not characteristic of Bulimia Nervosa.

    C.compensatory behavior is not a requirement for Binge Eating Disorder.

    D.a sense of a lack of control overeating during binges is not characteristic of Binge Eating Disorder.
    The correct answer is C.

    Compensatory behavior is a diagnostic criterion for Bulimia Nervosa but not for Binge Eating Disorder and, therefore, differentiates the two disorders.

    Answer A: Binges can be severe in both disorders and, therefore, is not a distinguishing characteristic.

    • Answer B: The disturbance in body image associated with Bulimia Nervosa involves self-evaluation that emphasizes body shape and weight.
    •   
    • Answer D: A sense of lack of control is characteristic of both disorders.
  18. For a diagnosis of Separation Anxiety Disorder, the DSM-5 requires symptoms of fear, anxiety, or avoidance that have lasted for at least __________ in children and adolescents or about __________ in adults.

    A.two weeks; four weeks

    B.four weeks; six months

    C.four weeks; six weeks

    D.two months; eight months
    The correct answer is B.

    The DSM-5 specifies a minimum duration of symptoms for Separation Anxiety Disorder of 4 weeks in children and adolescents and typically 6 months in adults.
  19. A DSM-5 diagnosis of Post-traumatic Stress Disorder or Acute Stress Disorder requires the development of characteristic symptoms following exposure to:

    A.actual or threatened death, serious injury, or sexual violence

    B.serious injury, or a threat to the physical integrity of self or others

    C.one or more severe psychosocial stressors

    D.a threat to the physical integrity of self or others
    The correct answer is A.

    Criteria for Post-traumatic Stress Disorder and Acute Stress Disorder include symptoms of intrusion and avoidance, as well as alterations in mood, cognition, and arousal following exposure to actual or threatened death, serious injury, or sexual violence.
  20. The core feature of the DSM-5 diagnosis of Gender Dysphoria is which of the following?

    A.Persistent preference for cross-gender roles

    B.Strong desire to be treated as the opposite gender

    C.Incongruence between assigned and experienced/expressed gender

    D.Cross-gender identification
    The correct answer is C.

    Incongruence between assigned gender and experienced/expressed gender is the core feature of Gender Dysphoria in the DSM-5.

    Answer A: A persistent preference for cross-gender roles in make-believe or fantasy play is one of the possible symptoms for children with Gender Dysphoria, but it is not the core feature.

    Answer B: The DSM-5 lists "a strong desire to be treated as the other gender (or some alternative gender different from one's assigned gender)" as one of the possible symptoms of Gender Dysphoria for adolescents or adults.

    Answer D: Cross-gender identification is not the core feature of this disorder, according to the DSM-5.
  21. The DSM-5 classifies symptoms of Substance Use Disorders into which of the following categories?

    A.Impaired control, social impairment, risky use, and pharmacological criteria

    B.Functional impairment, denial of consequences, impaired control, and pharmacological criteria

    C.Impaired control, tolerance, withdrawal, and functional impairment

    D.Functional impairment, tolerance and withdrawal, risky use, and marked distress
    The correct answer is A.

    The DSM-5 categorizes the indicators of problematic substance use in terms of the four categories listed in this answer. Impaired control refers to the tendency of the individual to use more of the substance than intended, inability to cut down on substance use, craving for the substance, and tendency to spend a great deal of time obtaining, using, or recovering from the effects of the substance. Social impairment refers to difficulties with social, occupational, and/or recreational activities due to substance use. Risky use refers to the use of the substance in situations that are potentially physically hazardous or despite knowing that physical or psychological problems may be exacerbated by substance use. Pharmacological criteria include tolerance and withdrawal.
  22. A DSM-5 diagnosis of Schizophrenia requires the presence of two or more active-phase symptoms during one month, with at least one symptom being ____________ plus continuous signs of disturbance for least six months.

    A.negative symptoms

    B.disorganized speech or disorganized behavior

    C.delusions or hallucinations

    D.delusions, hallucinations, or disorganized speech
    The correct answer is D.

    For a diagnosis of Schizophrenia, the DSM-5 specifies that at least one of the active-phase symptoms must be delusions, hallucinations, or disorganized speech.
  23. The intentional production of a physical symptom for the specific purpose of obtaining financial benefits is characteristic of which of the following?

    A.Malingering

    B.Factitious Disorder

    C.Conversion Disorder

    D.Somatic Symptom Disorder
    The correct answer is A.

    Malingering involves the intentional production of physical or psychological symptoms to obtain an external reward. The goal of obtaining an external reward distinguishes this condition from Somatic Symptom and Related Disorders.

    Answer B: Although Factitious Disorder involves the intentional production of symptoms, the goal is not to obtain an external reward.

    Answer C: Conversion Disorder involves somatic symptoms that are incompatible with recognized neurological or medical conditions.

    Answer D: Somatic Symptom Disorder involves somatic symptoms that are distressing with excessive thoughts, feelings, or behaviors related to the symptoms.
  24. A child with Oppositional Defiant Disorder is least likely to exhibit which of the following?

    A.Frequent conflicts with his/her parents

    B.A high degree of physical aggression toward peers

    C.Frequently blames others for his or her mistakes

    D.Often loses temper
    The correct answer is B.

    Oppositional Defiant Disorder (ODD) involves a pattern of angry/irritable mood, argumentative/defiant behavior, or vindictiveness. Although children with this disorder may be verbally aggressive, a high degree of physical aggression is not characteristic and may be a sign of comorbid Conduct Disorder.

    Answer A: It's likely a child diagnosed with ODD would exhibit conflict with their parents. 

    Answer C: Blaming is another behavior a child diagnosed with ODD may exhibit. 

    Answer D: Feeling angry/irritable or losing their temper is another symptom likely to be exhibited in a child diagnosed with ODD.
  25. The characteristic symptoms of Tobacco Withdrawal include all of the following except:

    A.irritability or anger

    B.hypersomnia

    C.impaired concentration

    D.increased appetite
    The correct answer is B.

    As described in the DSM-5, Tobacco Withdrawal involves the development of at least four characteristic symptoms within 24 hours of abrupt cessation or reduction in the use of tobacco - i.e., irritability or anger, anxiety, impaired concentration, increased appetite, restlessness, depressed mood, and/or insomnia.
  26. Psychological autopsies indicate that about ____% of people who complete suicide meet the criteria for one or more mental disorders at the time of their death.

    A.25

    B.55

    C.70

    D.90
    The correct answer is D.

    Psychological autopsy (PA) was originally developed as a method for identifying the mode of death in equivocal cases (i.e., homicide, suicide, natural causes, or accident); however, it has become the primary method for identifying the proximate cause of suicide. Based on a comprehensive review of PA studies, researchers concluded that approximately 90% of people who complete suicide had symptoms of at least one diagnosable mental disorder, and this conclusion has been confirmed by several subsequent studies (Psychology autopsy studies of suicide: a systematic review. Psychological Medicine, 33, 395-405, 2003).
  27. Cluster C Personality Disorders include which of the following Personality Disorders?

    A.Paranoid, Schizoid, and Schizotypal

    B.Schizoid, Antisocial, and Narcissistic

    C.Antisocial, Borderline, Histrionic, and Narcissistic

    D.Avoidant, Dependent, and Obsessive-Compulsive
    The correct answer is D.

    The Personality Disorders are divided into three clusters based on their similarities. Cluster C disorders involve anxiety or fearfulness and include Avoidant, Dependent, and Obsessive-Compulsive Personality Disorders.

    Answer A: These are Cluster A disorders, which involve odd or eccentric behaviors.

    Answer B: Schizoid Personality Disorder is a Cluster A disorder, while Antisocial and Narcissistic Personality Disorders fall within Cluster B.

    Answer C: These are Cluster B disorders, which are characterized by dramatic, emotional, or erratic behaviors.
  28. In comparison to bilateral ECT, unilateral ECT to the right (nondominant) hemisphere:

    A.produces similar levels of anterograde and retrograde amnesia

    B.produces greater anterograde amnesia for nonverbal tasks only

    C.produces greater anterograde and retrograde amnesia for both verbal and nonverbal tasks

    D.produces less anterograde amnesia for both verbal and nonverbal tasks
    The correct answer is D.

    Memory loss is considered one of the primary negative side-effects of ECT. However, there is evidence that memory loss with right unilateral ECT is less severe than with bilateral ECT. The part of the correct answer that mentions "amnesia for both verbal and nonverbal tasks" is distracting. The important information to note is that overall, the effects of unilateral ECT on ANY type of amnesia are less severe than those of bilateral ECT.
  29. Individuals who are diagnosed with Antisocial Personality Disorder fail to conform to social norms and:

    A.have a history of Conduct Disorder or Oppositional Defiant Disorder

    B.have a history of Conduct Disorder before 12 years of age

    C.have a history of Conduct Disorder before 15 years of age

    D.do not have a history of Conduct Disorder
    The correct answer is C.

    Individuals who are diagnosed with Antisocial Personality Disorder must be at least 18 years of and have a history of Conduct Disorder before 15 years of age.

    Answer A: The DSM-5 does not require a history of ODD for a diagnosis of Antisocial Personality Disorder; however, it does require a history of Conduct Disorder.

    Answer B and D: For a diagnosis of Antisocial Personality Disorder, Conduct symptoms must be present before 15 years of age.
  30. Research examining the relationship between mild depression and autobiographical memory has demonstrated that in comparison to the memories of people who are nondysphoric, the memories of people who are experiencing dysphoria are:

    A.less detailed and more negative

    B.more detailed and more negative

    C.less detailed and more neutral

    D.more detailed and more neutral
    The correct answer is A.

    Not surprisingly, studies have linked depression to an increased memory for negative events. People who are experiencing dysphoria are also likely to have autobiographical memories that are less detailed than those of people who are nondysphoric (Williams J.M., Scott J. Autobiographical memory in depression. Psychological Medicine. 1988 Aug;18(3):689-95).
  31. For Salvador Minuchin, "psychosomatic families" (e.g., those in which asthma, diabetes, or anorexia threaten the life of one child) are most likely to be characterized by which of the following?

    A.Frequent, intense, and open conflicts between family members

    B.weak boundaries between family members and limited opportunities for Individual autonomy

    C.Family roles that are inflexible and stereotyped

    D.Marked emotional distance between the husband and wife who are both emotionally immature
    The correct answer is B.

    Minuchin is associated with structural family therapy, which emphasizes the nature of the family structure including the boundaries between family members. According to Minuchin, diffuse (weak) boundaries are common in psychosomatic families.

    Answer A: According to Minuchin, psychosomatic families are characterized by a consistent avoidance of conflict.

    Answer C: Ackerman is more associated with the importance of roles in family dysfunction.

    Answer D: The concept of an "emotional divorce" is associated with Bowen.
  32. The ADDRESSING model:

    A.has received criticism for the lack of attention to factors related to ageism

    B.provides a framework for addressing cultural complexities in clinical practice

    C.excludes cultural competencies relevant to sexual orientation

    D.is a model specific for addressing the cultural complexities of people of color
    The correct answer is B.

    Pamela Hays is credited for developing the ADDRESSING framework, which provides a model for attending to cultural factors in therapy. Within this framework, culture is viewed as a multidimensional combination of Age, Developmental and acquired Disabilities, Religion, Ethnicity, Socioeconomic status, Sexual orientation, Indigenous heritage, National Origin, and Gender.

    Answer A: This is not true. In fact, the 'A' in the ADDRESSING framework is for age and generational influences.

    Answer C: The second 'S' in this acronym is for sexual orientation. The 'E' in the model is for ethnic/ racial identity.

    Answer D: The ADDRESSING framework is a model for attending to cultural factors in therapy; not specifically for addressing cultural complexities.
  33. Sue and Sue (1990) describe the worldview of counselors and their clients in terms of two dimensions - locus of control and locus of responsibility - and propose that members of racial minority groups are more likely to exhibit which of the following worldviews?

    A.Internal locus of control/internal locus of responsibility

    B.External locus of control/internal locus of responsibility

    C.Internal locus of control/external locus of responsibility

    D.External locus of control/external locus of responsibility
    The correct answer is C.

    D. W. Sue and D. Sue (Counseling the culturally different, New York, John Wiley, 1990) propose that an understanding of different worldviews and their impact adds to the effectiveness of cross-cultural counseling. Sue and Sue propose that minority group members are becoming increasingly aware of their own ethnic identity and the impact of racism on their lives - i.e., they are adopting a worldview that reflects an internal locus of control and an external locus of responsibility.

    Answer A: An internal locus of control/internal locus of responsibility is characteristic of the Westernized approach to counseling, white counselors, and white middle-class clients.

    Answer B: An external locus of control/internal locus of responsibility is characteristic of individuals who have been oppressed by the dominant group.

    Answer D: An external locus of control/external locus of responsibility is characteristic of a "placater" who adopts a passive role.
  34. You are conducting a therapy group and you are now in the middle stage. You ask group members to shift from answering direct questions to talking freely about themselves. The group members show resistance to this. You should:

    A.allow them to express their resistance

    B.explain to them the developmental process of the group

    C.tell them that resistance is not allowed

    D.go back to asking direct questions
    The correct answer is A.

    In group therapy, resistance can take several forms and occurs most often in the early middle stage of therapy when members try to establish their preferred position in the group. By definition, resistance is a covert effort to undermine change; thus, it is best handled by being brought out into the open. Discussions should focus on the here-and-now and deal with what the resistance implies.

    Answer B: While the developmental process of the group might be of interest to some members, this approach would do little, if anything, to deal with resistance.

    Answer C: This strategy would probably engender more intense resistance.

    Answer D: Capitulation to resistance amounts to reinforcement and is likely to increase its occurrence.
  35. During the first therapy session with a new client, a solution-focused therapist will:

    A.conduct a thorough screening and history-taking

    B.help the client identify and describe therapy goals in concrete, positive terms

    C.educate the client about the causes of his/her presenting problem and alternative techniques for alleviating the problem

    D.identify the client's stage of change
    The correct answer is B.

    As its name implies, solution-focused therapy focuses on solutions to problems rather than on the problems themselves. Stating goals in positive concrete (behavioral) terms helps the client recognize when he/she is accomplishing his/her goals and has been found to be a good predictor of outcome in solution-focused therapy.

    Answer A: This would occur during the intake session.

    • Answer C: Solution-focused therapists focus on solutions rather than problems.
    •   
    • Answer D: This sounds more like the transtheoretical model.
  36. A therapist tells a client who wants to quit smoking that he should keep his only pack of cigarettes in a place that is accessible only by walking for more than one mile. This is an example of which of the following paradoxical techniques?

    A.Reframing

    B.Symptom prescription

    C.Ordeal

    D.Restraining
    The correct answer is C.

    In this situation, smoking becomes self-punishing because of the amount of work (walking one mile) it will take in order to smoke. Haley (1984) described several types of ordeals. One type involves having the individual perform an unpleasant or inconvenient act in order to engage in the target behavior - which is what this question describes.

    Answer A: Reframing involves helping the client see a behavior in a different (and usually more positive) way.

    Answer B: Symptom prescription (aka prescribing the symptom) involves instructing the client to engage in the undesirable behavior in order to help the client recognize that the behavior is under his/her control.

    Answer D: Restraining involves encouraging the client not to change in order to elicit resistance and thereby facilitate change.
  37. In schools, psychological consultants are often hired to help teachers or other school personnel resolve problems related to specific individual students. This is referred to as:

    A.consultee-centered case consultation

    B.client-centered case consultation

    C.consultee-centered administrative consultation

    D.client-centered administrative consultation
    The correct answer is B.

    The question describes client-centered case consultation, which is the most common form of consultation in schools and elsewhere. In this situation, a consultant is working with a consultee to resolve a problem with a particular client (case).

    Answer A: In consultee-centered consultation, the focus is on the consultee's skills, abilities, and objectivity so that he/she can work more effectively with a particular group of clients in the future.

    Answer C: This type of consultation focuses on helping consultees (administrators) develop the skills they need to more effectively develop and implement programs.

    Answer D: This made-up term is not part of Caplan's model.
  38. Although the various models of homosexual (gay/lesbian) identity development differ somewhat in terms of the number and nature of the stages they propose, most describe a person in the initial stage as:

    revealing to another person that he/she is not sexually attracted to people of the opposite gender

    B.actively denying his/her sexual attraction toward same-gender individuals

    C.recognizing that he/she is different from same-gender peers

    D.first realizing that he/she is sexually attracted to people of the same gender
    The correct answer is C.

    You will want to be familiar with Troiden's (1988) homosexual identity development model for the exam. Troiden's initial stage is the sensitization stage and occurs prior to puberty. During this stage, the individual feels socially different from same-gender peers. For example, a girl may say that, unlike her female friends, she's not interested in boys or that she's more independent or aggressive than other girls; while a boy might say that he's less interested in sports and more interested in arts than his male peers.

    Answer A: Most models do not describe this as occurring during stage one.

    Answer B: This may occur during stage three.

    Answer D: Troiden described this as occurring during stage two.
  39. A client who is a member of a minority group expresses negative feelings about her own culture and positive feelings toward the dominant (Anglo) culture. The client is in which stage of Atkinson, Morten, and Sue's (1993) Racial/Cultural Identity Development Model?

    A.Conformity

    B.Dissonance

    C.Immersion

    D.Denial
    The correct answer is A.

    The Racial/Cultural Identity Development Model includes five stages: conformity; dissonance; resistance and immersion; introspection; and integrative awareness. Conformity is characterized by a preference for the dominant culture over one's own culture.

    Answer B: This stage is characterized by cultural conflict and confusion.

    Answer C: The immersion stage is marked by an active rejection of the dominant culture and acceptance of one's own culture.

    Answer C: Denial is not one of the stages of this model.
  40. Which of the following techniques would a psychologist most likely use to clarify the emotional distance or closeness between family members?

    A.sculpting

    B.mimesis

    C.psychodrama

    D.reframing
    The correct answer is A.

    In family sculpting, family members position themselves (or objects that represent them) in physical space in a way that reflects their relations and roles within the family system. The process usually involves having each family member, one at a time, create his or her own sculpture. Sculpting can be useful for revealing family members differing perceptions and feelings, especially each member's view of the emotional closeness or distance between them. Although sculpting is a very subjective technique, it often provides insight into how family members perceive their own and one another's roles and positions within the family.

    Answer B: Mimesis is associated with structural family therapy, in which the therapists first task is to develop a therapeutic system by joining the family in a position of leadership. Joining can be divided into three types: 1) Joining from a distant position: The therapist teaches and gives directive advice. 2) Joining from a median position (tracking): The therapist serves as an active, neutral listener, and uses techniques such as summarizing, observing, and commenting on processes. 3) Joining from a close position (mimesis): The therapist imitates (mimics) the affective range, style, and/or the content of the family's communications in order to build kinship with them and solidify the therapeutic alliance.

    Answer C: Psychodrama involves the use of dramatic techniques (such as role-reversal) through which clients are asked to act out past, present, or anticipated situations (usually socially stressful situations) and roles in order to gain new and deeper understanding and achieve catharsis.

    Answer D: Reframing is a verbal technique used to help clients change the meaning they give to an event, behavior, or experience by gently persuading them that it can be viewed in a different and more positive light.
  41. A therapy client's negative transference is most likely to be manifested in which of the following ways?

    A.The client will criticize the therapist and be pessimistic about the outcomes of therapy.

    B.The client will be more willing to talk about anxiety-arousing events.

    C.The client will say whatever comes to his/her mind.

    D.The client will express unrealistic positive expectations for the outcomes of therapy.
    The correct answer is A.

    The interpretation of a client's transferences is an important part of psychodynamic psychotherapy. Negative transference involves the projection of negative emotions onto the therapist and may manifest itself in several ways including making direct, negative comments about the effectiveness of therapy or being late for or missing appointments.

    Answer B: Transference is a form of resistance and, consequently, is not likely to increase a client's willingness to talk about anxiety-arousing events.

    Answer C: This describes the psychodynamic technique of free association and is not how a client would likely manifest transference.

    Answer D: Unrealistic positive expectations for the outcomes of therapy are more likely to be a manifestation of positive transference.
  42. As defined by Wrenn (1985), a "culturally encapsulated" counselor:

    A.has had little or no exposure to individuals from different cultures

    B.disregards cultural differences and his/her own cultural biases

    C.prioritizes culture as an etiological factor in mental illness

    D.recognizes that universal principles of behavior do not apply to everyone
    The correct answer is B.

    According to Wrenn, a culturally encapsulated counselor disregards cultural differences and defines everyone's reality according to his/her own cultural assumptions and stereotypes.

    Answer A: This may be true but it is not the best answer as it does not define cultural encapsulation.

    Answer C: Culturally encapsulated counselors do not prioritize but disregard cultural differences.

    Answer D: Culturally encapsulated counselors lack this awareness.
  43. From the perspective of Cross's (1991, 2001) Black Racial Identity Development Model, change in the direction of greater racial or ethnic awareness is:

    A.a natural consequence of a person's developmental level

    B.the result of exposure to events that increase the salience of race

    C.the result of peer influence

    D.one of the possible resolutions of an identity crisis
    The correct answer is B.

    Cross's model distinguishes between four stages of identity development: pre-encounter, encounter, immersion-emersion, and internalization. The stages of Cross's model are associated with different levels of race salience. For example, during the pre-encounter stage, race has low salience; but then exposure to one or more race-related events increases the salience of race and the individual progresses into the encounter stage.

    Answer A: Cross indicated that this awareness is caused by exposure to events rather than natural development.

    Answer C: Cross did not attribute this awareness to peer influence.

    Answer D: Cross did not attribute the development of this awareness to an identity crisis.
  44. Nonsexist therapy, in comparison to feminist therapy, places greater emphasis on:

    A.the impact of sex roles and sexism

    B.modification of personal behavior

    C.treating men and women the same

    D.the egalitarian nature of the therapeutic relationship
    The correct answer is B.

    • To a large degree, nonsexist therapy and feminist therapy overlap. Of the answers given, this is the best choice. While feminist therapy produces a change in personal behavior, that change is more consistently framed within the sociopolitical arena in which it occurs. In contrast, nonsexist therapy, while recognizing sociopolitical influences, also looks at personal change separately.
    •   
    • Answer A: This is characteristic of both forms of therapy.

    Answer C: Both types of therapy advocate that men and women be treated as equals but not necessarily the same.

    Answer D: This is an explicit goal of feminist therapy and, perhaps to a lesser degree, a characteristic of nonsexist therapy.
  45. Which of the following best describes the results of research on client and provider satisfaction with the use of videoconferencing and other forms of telehealth to deliver mental health services?

    A.Clients express high levels of satisfaction with telehealth but providers express low levels of satisfaction.

    B.Providers express high levels of satisfaction with telehealth but clients express low levels of satisfaction.

    C.Clients and providers both express satisfaction with telehealth and most clients express a stronger preference for telehealth than for in-person services.

    D.Clients and providers both express satisfaction with telehealth but some providers also express concerns about its effectiveness.
    The correct answer is D.

    Research on telehealth (which, in the context of mental health, is also known as telepsychology, telepsychiatry, and telemental health) has generally found that satisfaction ratings are high among both clients and providers. The research has focused primarily on clients and has found that they generally express high levels of satisfaction with telehealth. Studies looking at providers have confirmed that they also tend to be satisfied but that there is greater variability in their satisfaction ratings due to concerns about its effectiveness. See, e.g., J. Monnier, R. G. Knapp, and B. C. Frueh, Recent advances in telepsychiatry: An updated review, Psychiatric Services, 54, 1604-1609, 2003.

    Answer A: Both groups express satisfaction.

    Answer B: This has not been found to be true.

    Answer C: Although clients are satisfied with telehealth, they express similar levels of preference for telehealth and in-person services.
  46. A client is acting in a passive-aggressive way toward a psychology intern who, in turn, becomes unconsciously passive-aggressive toward her supervisor. This is an example of which of the following?

    A.Transference

    B.Countertransference

    C.Over-involvement

    D.Parallel process
    The correct answer is D.

    In this situation, the intern's behavior with the counselor is "mirroring" the client's behavior with the intern. This type of mirroring is referred to as parallel process.

    Answers A & B: Parallel process is conceptualized as a reflection of the transference/countertransference process.

    Answer C: This is a made-up term.
  47. The term homophobia was coined by:

    A.Richard Troiden

    B.Gregory Herek

    C.George Weinberg

    D.Claire Renzetti
    The correct answer is C.

    George Weinberg was a Jewish-American psychologist who termed the phrase homophobia in the 1960's.

    Answer A: Richard Troiden was a researcher who proposed the Homosexual Identity Development Model.

    Answer B: Gregory Herek opposed the use of the word homophobia as he believed this term was ambiguous and imprecise.

    Answer D: You may recognize Renzetti's name from the study materials which include her theorizations about internalized homophobia.
  48. For an extended family systems (Bowenian) therapist, when a family exhibits a high degree of fusion but one member is more differentiated than the others, the likely approach would be to:

    A.work with the least differentiated family members

    B.work with the most differentiated family member

    C.work with all members of the extended family as a group

    D.work with all family members in individual therapy until they reach a similar level of differentiation
    The correct answer is B.

    According to Murray Bowen, a high degree of fusion (emotional "stuck togetherness") is the source of family dysfunction. The goal of Bowen's form of family therapy, therefore, is to help family members become less fused, or more differentiated (individuated). In contrast to many other family therapists, Bowen's form of therapy typically does not involve seeing all of the family members. Bowen often worked with the most differentiated family member on the assumption that, when that member changed in a positive way, his/her change would motivate other family members to move toward greater differentiation.

    Answer A: Followers of Bowen would likely work first with the most differentiated family member.

    Answer C: In contrast to many other family therapists, Bowen typically worked with the two most significant adult family members. This typically involved working with the parents or the parent who was the most differentiated, even when the "identified patient" was a child.

    Answer D: This is not a technique used by Bowen.
  49. Harry Stack Sullivan's interpersonal approach to psychiatry identifies ____________ as the primary cause of psychopathology.

    A.unconscious conflicts manifested into present behavior 

    B.excessive anxiety

    C.obstacles to self-actualization

    D.avoidance of ultimate concerns
    The correct answer is B.

    Sullivan believed that human behavior is motivated by two needs - the need for satisfaction and the need for security. The need for satisfaction is fulfilled by things that meet the individual's biological needs (food, water, shelter, etc.), while the need for security is fulfilled by gratifying experiences with others. Sullivan emphasized the role of anxiety in personality development and psychopathology. As described by Sullivan, excessive anxiety is the result of interpersonal insecurity that can be traced to problems in interpersonal relationships (especially during infancy and childhood) and is the basis for most psychiatric problems.

    Answer A: This is more in line with psychodynamic theory.

    Answer C: This is a premise of person-centered therapy.

    Answer D: This sounds more like existential therapy.
  50. Which of the following terms is least descriptive of the Lockean tradition underlying the Western approach to individual psychotherapy?

    A.Linear-cause effect

    B.Reductionistic

    C.Individualistic

    D.Relativistic
    The correct answer is D.

    Most individual therapies - e.g., Freudian psychoanalysis, Jung's analytical psychology, Adler's individual psychology, and Rogerian therapy - reflect a Lockean approach. A Lockean approach emphasizes linear cause-effect relationships, individualism over collectivism, and a deterministic and reductionist perspective. Relativism is more characteristic of non-western and systems approaches. The Lockean tradition reflects an absolutist approach.
  51. An emphasis on contact, awareness, and experimentation is central to which of the following?

    A.Gestalt therapy

    B.Solution-focused therapy

    C.Adlerian therapy

    D.Rational-emotive therapy
    The correct answer is A.

    Knowing that the goal of Gestalt therapy is to achieve conscious awareness would help to identify the correct response. Contact, conscious awareness, and experimentation are key targets in Gestalt therapy. Contact refers to being in touch with what is happening in the here-and-now; awareness refers to focused attention in situations requiring it; and experimentation is the act of trying something new in order to increase understanding.

    Answer B: The emphasis of solution-focused is a focus on solutions versus problems.

    Answer C: The focus of Adlerian includes: inferiority feelings, striving for superiority, style of life, and social interest.

    Answer D: In Rational-emotive therapy an emphasis is placed on challenging irrational beliefs that lead to negative emotional consequences, which result in self-defeating behaviors.
  52. For Carl Jung, transference is:

    A.a form of projection

    B.sexual "acting out"

    C.symbolization

    D.a fantasy
    The correct answer is A.

    For Jung, transference is a specific form of the more general process of projection. Jung believed that, when transference occurs in therapy, elements of the client's personal and collective (transpersonal) unconscious are projected onto the therapist.

    Answer B: Jung disagreed with Freud's belief that transferences are inherently sexual in nature.

    Answer C: Although symbols are an important part of Jung's theory, he did not describe transference as "symbolization."

    Answer D: Jung did not describe transference as a form of fantasy.
  53. In terms of group therapy, premature termination has been found to depend most on the members':

    A.locus of control

    B.expectations

    C.psychological mindedness

    D.diagnosis
    The correct answer is B.

    One of the most consistent findings of the research on group therapy is that people who terminate prematurely from group treatment are those who have unrealistic expectations and unfavorable attitudes.
  54. Sue et al. (1991) found that ethnic matching was associated with improved treatment outcomes for:

    A.Hispanic clients

    B.Black clients

    C.Asian clients

    D.Clients of all races
    The correct answer is A.

    Sue et al (1991) found a correlation between ethnic matching and improved treatment outcomes for Hispanic clients only.

    Answers B: Sue et al. (1991) did not find a correlation between ethnic matching and treatment outcomes for Black clients.

    Answer C: The researchers of this study found a correlation between ethnic matching and reduction of premature termination for Asian clients; however, ethnic matching was not found to improve treatment outcomes for this population.

    Answer D: The researchers of this study only found a correlation between ethnic matching and treatment outcomes for Hispanic clients.
  55. An African American adult in which stage of Cross's (1991) Black Racial Identity Development Model denigrates all aspects of White culture while idealizing all aspects of African American culture?

    A.Internalization

    B.Encounter

    C.Immersion-emersion

    D.Reintegration
    The correct answer is C.

    Cross distinguishes between four stages of identity development: pre-encounter, encounter, immersion-emersion, and internalization. During the immersion-emersion stage, the individual withdraws from the dominant (White) culture and becomes immersed in African American culture.

    Answer A: A person in the internalization stage has adopted an African American worldview but is flexible in his/her attitudes toward Whites.

    Answer B: During the encounter stage, the individual becomes increasingly aware of his/her racial heritage.

    Answer D: Reintegration is not one of the stages included in Cross's model.
  56. Structural family therapy entails three overlapping steps or stages. Which of the following techniques is/are most useful during the first step?

    A.Reframing and relabeling

    B.Tracking and mimesis

    C.Constructing a family map

    D.Enactment
    The correct answer is B.

    The three steps in structural family therapy are joining, evaluating/diagnosing, and restructuring. Tracking (identifying and using the family's values, life themes, etc.) and mimesis (adopting the family's affective and behavioral style) are methods used to join the family system. Joining is the initial step in structural family therapy.

    Answer A: Reframing and relabeling are restructuring techniques.

    Answer C: A family map is used to facilitate the structural diagnosis of the family.

    Answer D: Enactment is a technique used to facilitate diagnosis and restructuring of the family.
  57. Dr. Stilwell receives a letter from the Ethics Committee stating that the Committee has received a complaint from a current client who is concerned about Dr. Stilwell's office staff and procedures. The client has found two members of the staff to be consistently rude, and they have made comments to her that imply they have access to confidential information. Dr. Stilwell should _______________________.
    The correct answer is D.

    This question is difficult to answer because none of the responses describes an entirely inappropriate action. However, only this response addresses the APA's Ethics Code and Rules and Procedures. Both documents require psychologists to cooperate with the Ethics Committee. Specifically, Part II, Standard 8.2 of the Rules and Procedures requires psychologists to respond to charges of unethical conduct personally, and Standard 9 requires full cooperation "in a timely fashion." How Dr. Stilwell responds to the client's charges - e.g., by talking to the client, discussing the matter with the staff - depends on the situation.

    Answer A: This answer is incorrect because it fails to address the more central issue of responding to ethical complaints filed with the Ethics Committee.

    Answer B: While this action may be warranted, this is not the best answer.

    Answer C: Dr. Stilwell's response will likely depend on the circumstances of this case.
  58. Dr. Wright, a licensed psychologist, has been seeing Aimee Jacobs in individual therapy for nearly six months. Aimee has just decided to divorce her husband and asks Dr. Wright if he will conduct custody evaluations of her three children. As an ethical psychologist, Dr. Wright ________________.
    The correct answer is D.

    This issue is addressed in Paragraph 7 of the Guidelines for Child Custody Evaluations in Family Law Proceedings (APA, 2010), which states that "psychologists strive to avoid conflicts of interest and multiple relationships in conducting evaluations."

    Answer A: Conducting an evaluation after working as a therapist with the parents would create a complex multiple relationship that should be avoided.

    Answer B: This answer is incorrect because it creates problematic multiple roles for Dr. Wright.

    Answer C: Given the circumstances, it is highly unlikely Dr. Wright could be objective in conducting a custody evaluation.
  59. A licensed psychologist who provides individual and group therapy to adolescents and adults learns she has an aggressive form of cancer. In terms of ethical requirements, the psychologist __________.

    A.should refrain from initiating any professional activities that might be adversely affected by her medical condition

    B.should obtain supervision to ensure that her medical condition does not impair her ability to provide effective services to clients

    C.should inform her clients of her medical condition "as early as is feasible"

    D.is not obligated to take any special actions or precautions in this situation
    The correct answer is A.

    Standard 2.06 of the Ethics Code requires psychologists to "refrain from initiating an activity when they know or should know there is a substantial likelihood that their personal problems will prevent them from performing their work-related activities in a competent manner." Personal problems include emotional, social, health-related, and other personal issues.

    Answer B: Although the psychologist should "take appropriate measures" if she believes her condition may interfere with the performance of her work-related duties, supervision may not be the best course of action.

    Answer C: It is not necessary for the psychologist to inform her clients of her condition and, in some cases, may not be in the best interests of the clients to do so.

    Answer D: This answer does not align with Section 2.06 of the Ethics Code.
  60. A psychologist acts as both a consultant to the court and a fact witness for the defense in a criminal case. This __________________________.

    A.represents a clear conflict of interest and is prohibited by APA guidelines

    B.represents a "multiple relationship" and is explicitly prohibited by APA guidelines

    C.may be acceptable as long as the psychologist clarifies her roles with the court and the defense

    D.may be acceptable as long as the psychologist gets written permission from the court to act as a fact witness
    The correct answer is C.

    Dual (multiple) relationships in forensic settings are addressed in the Ethics Code and the Specialty Guidelines for Forensic Psychology. Accepting dual roles, such as acting as a consultant for one party and a fact witness for another, may be acceptable as long as the psychologist clarifies his/her roles with both parties and acts in a way that does not compromise his/her judgment and objectivity.

    Answer A: Although a conflict of interest should always be avoided, this particular situation does not necessarily constitute a conflict of interest.

    Answer B: This situation may be described as a "multiple relationship," but it is not one that is explicitly prohibited by APA guidelines.

    Answer D: Written permission from the court is not necessary.
  61. A client reveals to you that he and a friend robbed a convenience store six months ago. He says he knows that what he did was wrong and will never do it again. He also says that he notices that you "make notes" while he is talking, and he asks that you not keep any written record of the robbery. You should ____________________________.

    A.agree to exclude information about the robbery from his file

    B.determine whether the robbery is clinically relevant before deciding whether or not to include information about it in his file

    C.inform the client that you are legally required to report the matter to the authorities before you actually do so

    D.reassure the client that anything he tells you is confidential and will not be revealed without his consent
    The correct answer is B.

    Psychologists are generally not required to report criminal conduct unless it is conduct explicitly addressed by the law (e.g., child abuse, impending danger to others). This is the best response of those given. A similar situation is discussed by R. Woody, who notes that client records are for the benefit of both the client and the therapist and that, if the client "communicates information that is relevant to the services provided, it should be entered into the record" (Legally safe mental health practice: Psycholegal questions and answers, Madison, CT, Psychosocial Press, 1997, p. 70).

    Answer A: Omitting client information from the record at the client's request is not necessarily in the best interests of the client or the therapist.

    Answer C: Psychologists do not generally report past criminal conduct unless it involves child or elder abuse.

    Answer D: It is important to keep in mind that it is impossible to guarantee that any client information will never be revealed under any circumstance.
  62. The police arrive at your office with an arrest warrant for one of your clients. They tell you that they need the client's most recent address and telephone number. You should _____________________.

    A.provide them with the information they request

    B.make a copy of the warrant for your files before providing them with the information they request

    C.tell them you will have to get a signed waiver from the client before you can release the information

    D.not give them any information about the client
    The correct answer is D.

    An arrest warrant does not constitute an exception to the therapist-client privilege. In order to provide any information about the client, including the fact that he/she is a client, a signed release or a court order would be necessary.

    Answer A: A psychologist should not provide any information without a release.

    Answer B: A psychologist should not provide any information without a release, including whether or not a person is a client.

    Answer C: Without a release from the client, the psychologist should not indicate whether a person is a client.
  63. During her first therapy session, a client tells you that she wants to pay for her sessions in cash and that she doesn't want you to keep a record of her sessions or payments. If you agree to this arrangement _______________________.

    A.you are acting ethically as long as you ascertain that her reason for making this request is valid

    B.you are acting ethically since it is up to you to decide what to include in a client's records

    C.you are acting ethically and legally as long as you report her payments as income on your income tax forms

    D.you are acting unethically and possibly illegally
    The correct answer is D.

    This answer is most consistent with legal and ethical requirements. For example, Guideline 2 of the APA's (2007) Record Keeping Guidelines states that a "psychologist strives to maintain accurate, current, and pertinent records of professional services as appropriate to the circumstances and may be required by the psychologist's jurisdiction." Standard 6.01 of the APA's (2002) Ethic Code presents reasons why it is important for psychologists to create and maintain adequate records.

    Answer A: Accurate record keeping is an ethical and legal requirement.

    Answer B: The APA publishes Record Keeping Guidelines that specify accepted standards for record keeping.

    Answer C: Financial record-keeping is a different matter than clinical record-keeping, and this answer does not address the need to maintain clinical notes.
  64. "Vicarious liability" is most likely to be of concern when a psychologist is acting in the role of _______________________.

    A.collaborator

    B.consultant

    C.advocate

    D.supervisor
    The correct answer is D.

    The doctrine of vicarious liability ("respondeat superior") applies to settings in which a psychologist has ethical and legal responsibility for the services provided by another person. Ethical and legal responsibility is most likely to be an issue in a supervisory relationship in which there is a large discrepancy between the skills and knowledge of the supervisor and those of the supervisee. Vicarious liability can also be an issue in employer-employee relationships.

    Answer A: A collaborator is not in a position of authority or control and therefore has no vicarious liability.

    Answer B: Because a consultant is not exercising power over another person's decision-making, there is no vicarious liability.

    Answer C: There is no vicarious liability when a psychologist is serving as an advocate.
  65. According to the APA's "Guidelines for Psychological Services to Ethnic, Linguistic, and Culturally Diverse Populations," when a client belonging to a different ethnic or cultural group comes to therapy, a therapist is best advised to _____________________.

    A.treat the client like any other person unless there is reason to do otherwise

    B.alter the diagnostic and treatment processes to coincide with the client's ethnic or cultural background

    C.recognize that ethnic and cultural forces can have an impact on both the therapist's and the client's psychological processes

    D.refer the client to another therapist from the same ethnic or cultural background whenever it is feasible to do so
    The correct answer is C.

    The "Guidelines for Psychological Services to Ethnic, Linguistic, and Culturally Diverse Populations" addresses several issues related to assessment, intervention, and research with clients from different ethnic and cultural backgrounds. Paragraph 3a states that "Psychologists recognize ethnicity and culture as significant parameters underlying psychological processes," and this refers not only to the processes of the client but also to those of the therapist. Therapists must be aware of their own prejudices and stereotypes when working with clients from different ethnic and cultural groups.

    Answer A: Although there are times when this would be the appropriate course of action (e.g., when the client exhibits a high degree of acculturation into the "mainstream" culture), it is not a recommended strategy.

    Answer B: This may be appropriate in some situations, but not all of them.

    Answer D: This may be appropriate in some situations but is not a general requirement.
  66. With regard to informed consents in research, APA's Ethics Code ___________________.

    A.requires psychologists to provide potential research participants with specific information in "reasonably understandable language" prior to consenting to participate

    B.requires psychologists to "ensure that potential research participants understand" the information they have been given prior to consenting to participate

    C.requires psychologists to provide potential research participants with specific information prior to consenting to participate

    D.requires psychologists to provide potential research participants with specific information in "reasonably understandable language" and to ensure that they understand the information prior to consenting to participate
    The correct answer is A.

    With certain exceptions, psychologists must obtain informed consent from individuals prior to their participation in a research study. The issue of language and informed consent is addressed in Standard 3.10 of the Ethics Code. It outlines the specific information that psychologists must give potential participants, therapy clients, and others as part of the informed consent process and states that the information must be supplied in "reasonably understandable language."

    Answer B: The Code does not explicitly state that psychologists must ensure that participants understand the information.

    Answer C: This answer is incorrect because it lacks the specific language contained in Answer A.

    Answer D: The second half of this answer that deals with making sure participants understand the information is not required.
  67. According to the APA's "General Guidelines for Providers of Psychological Services," a "user" of clinical and counseling psychological services includes _______________________________.

    A.direct users of services only ("patients" and "clients")

    B.direct users and third-party purchasers of such services

    C.direct users, third-party purchasers, and sanctioners (people who have legitimate concern related to the provision of psychological services)

    D.direct users, third-party purchasers, sanctioners, and public/private facilities and organizations receiving psychological services
    The correct answer is D.

    According to the General Guidelines, a "user" of clinical and counseling psychological services includes direct users, third-party purchasers, sanctioners, and public/private facilities and organizations receiving psychological services.

    Answer A: The users of psychological services are not limited to direct users only.

    Answer B: The users of psychological services are not limited to direct users and third-party purchasers only.

    Answer C: The users of psychological services are not limited to direct users, third-party purchasers, and sanctioners only.
  68. The best predictor of whether a therapist and client will become sexually involved is ________________.

    A.the client's level of physical attractiveness

    B.the client's history of sexual abuse

    C.the therapist's professional experience and status

    D.the therapist's past history with regard to sexual involvement with a client
    The correct answer is D.

    Research shows that recidivism rates for sexual misconduct among therapists are quite high, leading to the conclusion that a therapist's history with regard to sexual involvement with clients is the best predictor of future involvement.

    Answer A: The client's physical appearance is not a factor in sexual misconduct among therapists.

    Answer B: Although certain client behaviors and background factors may place a client at higher risk for sexual involvement with a therapist, client characteristics are not as good as the therapist's history at predicting whether or not the client will become sexually involved with his/her therapist.

    Answer C: The therapist's experience and status have not been found to correlate highly with sexual misconduct.
  69. Dr. Goodman has been dating Reginald for seven weeks when she realizes that he is the brother of a client she has been seeing in therapy for nearly three months. Dr. Goodman's best course of action would be to _____________.

    A.refer the client to another therapist immediately

    B.stop seeing Reginald immediately

    C.discuss the matter with the client as soon as possible

    D.do nothing until it becomes evident that the situation is creating a conflict
    The correct answer is C.

    The APA's Ethics Code warns against becoming involved in multiple relationships. This response is most consistent with the spirit of Standard 3.05(b) of the Ethics Code which states, "If a psychologist finds that, due to unforeseen factors, a potentially harmful multiple relationship has arisen, the psychologist takes reasonable steps to resolve it with due regard for the best interests of the affected person and maximal compliance with the Ethics Code."

    Answer A: It may be necessary, eventually, to refer the client to another therapist, but this answer is not the best initial action.

    Answer B: While it may be necessary, at some point, to end the relationship with Reginald, this answer is not the best initial action.

    Answer D: Dr. Goodman should not wait to take action.
  70. The APA's (2007) Record Keeping Guidelines recommends that, in the absence of superseding laws or regulations, psychologists should retain __________.

    A.the full record of an adult client until seven years after termination of therapy

    B.the full record of an adult client until 12 years after termination of therapy

    C.the full record of an adult client until three years after termination and a summary of the record for at least four additional years

    D.the full record of an adult client until five years after termination and a summary of the record for at least seven additional years
    The correct answer is A.

    Paragraph 7 of the 2007 version of the APA's Record Keeping Guidelines specifies that a "psychologist strives to be aware of applicable laws and regulations and to retain records for the period required by legal, regulatory, institutional, and ethical requirements." In comments on the application of Paragraph 7, the Guidelines states that, in the absence of superseding laws or institutional regulations, "psychologists may consider retaining full records until 7 years after the last date of service delivery for adults or until 3 years after a minor reaches the age of majority, whichever is later. In some circumstances, the psychologist may wish to keep records for a longer period, weighing the risks associated with obsolete or outdated information, or privacy loss, versus potential benefits associated with preserving the records.
  71. The Ethics Code requires that a psychologist discuss the issue of fees and billing policies with a new client ____________________.

    A.prior to the first consultation

    B.during the first session

    C.when the psychologist deems it appropriate

    D.as early as is feasible
    The correct answer is D.

    This issue is addressed in Standard 6.04(a): Fees and Financial Arrangements. "As early as is feasible" is the language used in the Ethics Code and, therefore, is the best answer to this question.

    Answer A: This answer does not reflect the language used in the Ethics Code.

    Answer B: While the first session is the time when many psychologists discuss fees with clients, the Ethics Code does not offer that degree of specificity.

    Answer C: This answer is incorrect because it does not capture the language used in the Ethics Code.
  72. Which of the following statements is most consistent with the requirements of the APA's Ethics Code with regard to the use of deception in research?

    A.Psychologists may not deceive participants about possible risks that may affect their willingness to participate.

    B.Psychologists are not required to disclose any possible risks if they have taken all possible steps to reduce those risks.

    C.Psychologists must provide an explanation of the use of any deception in an experiment immediately after participation.

    D.Psychologists may not use deception in research.
    A.Psychologists may not deceive participants about possible risks that may affect their willingness to participate.B.Psychologists are not required to disclose any possible risks if they have taken all possible steps to reduce those risks.C.Psychologists must provide an explanation of the use of any deception in an experiment immediately after participation.D.Psychologists may not use deception in research.
  73. Dr. Lin is hired by a company to evaluate an employee for promotion into a high-level position. The company manager tells Dr. Lin that the employee was evaluated six years ago prior to his last promotion, and the company wants Dr. Lin to make a recommendation based on those results without an additional evaluation. Dr. Lin is concerned that some of the information from the evaluation is no longer relevant. He should ___________.

    A.comply with the manager's request since it is up to her (the manager) to decide what is relevant

    B.comply with the manager's request but warn her of any potential limitations of her recommendation

    C.comply with the manager's request only if the employee agrees to the conditions

    D.discuss with the manager the possibility that the data are obsolete and that new data should be collected
    The correct answer is D.

    This question is pretty straightforward. It is unethical to base decisions on obsolete data. Psychologists have a responsibility to ensure that their recommendations are accurate, which would not be the case if they rely on obsolete data.

    Answer A: This answer is incorrect because it does not reflect the psychologist's obligations.

    Answer B: Providing information about the possible limitations of a recommendation would be insufficient.

    Answer C: The employee's permission, in this case, is less relevant than the issue of outdated results.
  74. The brother of a former client of yours calls you requesting that you release the client's therapy records to him. The client died several months prior to this request. As an ethical psychologist, you _______________________.

    A.release the records to him since he is a family member

    B.release the records to him since requirements for maintaining confidentiality do not extend beyond a client's death

    C.release the records only after obtaining a waiver from the executor or administrator of the client's estate

    D.refuse to release the records under any circumstance
    The correct answer is C.

    On the exam, it is best to choose the response that protects the client's welfare. Although this situation is not explicitly addressed by the Ethics Code, Standard 4.05(a) states that "psychologists may disclose information with the appropriate consent of the organizational client, the individual client/patient, or another legally authorized person on behalf of the client/patient unless prohibited by law. This response is most consistent with ethical requirements as well as with state laws. For example, a Massachusetts statute relevant to the practice of psychology states that "all communications ... shall be deemed to be treated as confidential in perpetuity" (251 CMR 1.11). And, in jurisdictions that have similar laws, the records of a deceased client cannot be released to a family member or other person without a court order or release from the executor or administrator of the client's estate.

    Answer A: The psychologist's responsibility to maintain confidentiality does not terminate upon a client's death.

    Answer B: When a client has died, psychologists continue to protect client confidentiality to the degree permitted by law. Most states require that patient records remain confidential after the patient's death.

    Answer D: According to APA's Ethics Code Standard 4.05(a), a psychologist may release information with the consent of a "...legally authorized person on behalf of the client/patient...."
  75. A new client of yours, who is gay, tells you that he overheard his previous therapist make derogatory remarks about his sexual orientation to a co-worker. Your best course of action would be to _____________________________.

    A.discuss his option of filing a complaint against the psychologist with the Ethics Committee

    B.file a complaint against the psychologist with the Ethics Committee yourself

    C.call the psychologist and tell him that your client overheard his remarks

    D.provide the client with support and remind him that there are many prejudiced people in the world
    The correct answer is A.

    This is a difficult question that requires a careful reading of the responses. Discussing the option of filing a complaint is usually an acceptable course of action when a client reveals that a previous therapist has acted unethically.

    Answer B: This answer is incorrect because it does not address the issue of the client's confidentiality. Before contacting the Ethics Committee, you would want to get a waiver from the client.

    Answer C: This answer is incorrect, as it fails to address the issue of the client's confidentiality.

    Answer D: The better response entails discussing with the client his options.
  76. A 13-year old client tells his therapist that he wants to kill himself and that he has access to his father's gun collection. The therapist believes the boy's threat is a serious one and tells him that she will have to contact his parents. The boy gets very angry and says he thought they had agreed that everything he said in therapy was confidential. If the therapist contacts the boy's parents, she has ____________________________.


    A.acted ethically only if the limits of confidentiality were discussed with the boy at the beginning of therapy

    B.acted ethically because a breach of confidentiality is justified in this situation

    C.acted unethically if confidentiality was, in fact, guaranteed to the boy at the outset of therapy

    D.acted unethically if she contacts the boy's parents without his consent
    The correct answer is B.

    A psychologist must take appropriate action whenever a client is a danger to him/herself. Contacting the boy's parents would be an appropriate action. In this situation, a breach of confidentiality is both ethical and legal.

    Answer A: While the limits of confidentiality should have been discussed in advance, this answer fails to address the more central concern, which is that the psychologist needs to take appropriate action in light of the suicide threat.

    Answer C: Although a psychologist should not guarantee confidentiality under all circumstances, this answer fails to address the more critical concern of danger.

    Answer D: This answer is incorrect because the suicide threat requires the psychologist to act on the client's behalf.
  77. You are asked to assist with a single-parent therapy group. The therapist leading the group is also conducting a study designed to evaluate the psychological problems of parents who are raising their children alone. You become aware that the therapist has not informed group participants that they do not have to be included as subjects in the research project. Your first action should be to:

    A.immediately discontinue your participation in the group

    B.advise participants that they may withdraw from the group

    C.advise the therapist that he is acting unethically

    D.report the therapist to the local ethics committee
    The correct answer is C.

    APA's Ethics Code requires that research participants be informed that they may withdraw at any time from the research. Thus, the therapist leading the group is acting unethically. In terms of response to an ethical violation, the Ethics Code recommends that a psychologist first attempt an informal resolution by discussing the matter with the violator. Thus, this response is most consistent with the provisions of the Ethics Code.

    Answer A: Addressing the ethical violation directly with the other therapist is the best answer.

    Answer B: Ethics Guidelines advise psychologists to address the violation with the other professional when possible.

    Answer D: Addressing the other therapist first is the correct action.
  78. At the end of her first therapy session, a middle-aged woman tells you she is presently receiving treatment from a psychiatrist. You, as an ethical psychologist, should ____________________________.

    A.inform the woman that it would be unethical to continue seeing her while she is seeing another professional

    B.call the psychiatrist to inform him of the situation and obtain his permission to continue therapy with her

    C.discuss the issues related to the woman's relationship with the psychiatrist during your next session with her

    D.allow the woman to decide when to terminate treatment with the psychiatrist
    The correct answer is C.

    Standard 10.04 of the Ethics Code states, "In deciding whether to offer or provide services to those already receiving mental health services elsewhere, psychologists carefully consider the treatment issues and the potential client's/patient's welfare. Psychologists discuss these issues with the client/patient or another legally authorized person on behalf of the client/patient in order to minimize the risk of confusion and conflict, consult with the other service providers when appropriate, and proceed with caution and sensitivity to the therapeutic issues."

    Answer A: Accepting the woman for treatment in this situation is not prohibited by the Ethics Code.

    Answer B: Obtaining the permission of the psychiatrist to see the woman is not required by the Ethics Code and would violate the client's confidentiality if it is done without her consent.

    Answer D: This answer is incorrect, as it is less consistent with Standard 10.04.
  79. You are starting a new program for children of recently divorced parents. You tell the editor of the local newspaper, who has just divorced her husband, that her two children can attend the program for free if she puts an article about the program in the paper. According to the Ethics Code in regards to the press, this is _____________________.

    A.unethical because it constitutes coercion

    B.unethical because psychologists are prohibited from compensating a newspaper employee for publicity in a news item

    C.unethical only if you influence the content of the article

    D.ethical as long as the arrangement doesn't compromise your objectivity when working with the editor's children
    The correct answer is B.

    This issue is addressed in Standard 5.02(b) of the APA's Ethics Code which states, "Psychologists do not compensate employees of press, radio, television, or other communication media in return for publicity in a news item."

    Answer A: One could argue the arrangement is coercive in nature and blurs professional boundaries, but the primary goal of this question is to test whether candidates know psychologists are prohibited from compensating employees of the press for providing publicity in a news item.

    Answer C: Paying the newspaper employer for a story, regardless of content, is unethical.

    Answer D: This answer is incorrect, as Standard 5.02(b) is clear on this issue.
  80. You are meeting with a teenager suffering from an eating disorder for individual treatment. At the end of that first session, her parents and siblings join in the session as agreed upon with the client. You should _________________________.

    A.obtain a release of information from all family members

    B.clarify at the outset your relationship with each family member

    C.refer the daughter who is the identified patient for individual therapy

    D.include only the parents and identified patient in family therapy
    The correct answer is B.

    According to Standard 10.02(a) of the APA's Ethics Code, "When psychologists agree to provide services to several persons who have a relationship (such as spouses, significant others, or parents and children), they take reasonable steps to clarify at the outset (1) which of the individuals are clients/patients and (2) the relationship the psychologist will have with each person. This clarification includes the psychologist's role and the probable uses of the services provided or the information obtained."

    Answer A: A release of information is not necessary.

    Answer C: While a referral ultimately may be appropriate, this answer misses the more salient point that the therapist needs to clarify the relationship she or he will have with each party.

    Answer D: While the therapist may ultimately decide to limit ongoing therapy to specific family members, this answer fails to account for the fact that the psychologist needs to clarify the relationship she or he will maintain with each member at the outset.
  81. Providing therapy to a former sex partner ____________________.

    A.is not directly addressed by the Ethics Code but is covered by the prohibition against multiple relationships

    B.is permissible only when it is not clinically contraindicated and the chance of exploitation is minimal

    C.is permissible under "unusual circumstances" when at least two years have passed since the sexual relationship ended

    D.is explicitly prohibited by the Code
    The correct answer is D.

    Standard 10.07 of the Ethics Code addresses the issue of therapy with former sexual partners. According to the Code, "Psychologists do not accept as therapy clients/patients persons with whom they have engaged in sexual intimacies." There are no exceptions to this prohibition.

    Answer A: The code explicitly addresses sexual relationships with clients.

    Answer B: The provisions of Standard 10.07 of the Ethics Code do not include exceptions. Psychologists do not accept as patients individuals with whom they have had sexual relationships.

    Answer C: If you picked this response, you've mixed up the situation asked about in this question with becoming involved sexually with a former client.
  82. You used a single standardized personality inventory to assess a defendant who is accused of rape. The defendant's profile does not seem to be consistent with the nature of the crime. You _______________________________.

    A.should testify as an expert witness that you believe the defendant is not guilty

    B.should testify as a character witness

    C.should not disclose the results of this personality inventory

    D.should not base a conclusion on the results of a single assessment profile
    The correct answer is D.

    According to Standard 9.01(a) of the APA's Ethics Code, "psychologists base the opinions contained in their recommendations, reports, and diagnostic or evaluative statements, including forensic testimony, on information and techniques sufficient to substantiate their findings." Consequently, you should not draw a conclusion in this situation based solely on the results of a single assessment test.

    Answer A: Basing a conclusion on the results from a single instrument represents poor practice.

    Answer B: This answer is incorrect because it fails to address the more central issue of making decisions based on adequate assessment information.

    Answer C: A single assessment instrument does not provide enough information on which to base findings in this case.
  83. Responsibility for the validity of information provided by an automated test scoring and interpretation service lies with ___________________________.

    A.the test scoring and interpretation service

    B.the psychologist who is in charge of the test scoring and interpretation service

    C.the psychologist who uses the test scoring and interpretation service

    D.the test scoring and interpretation service and the psychologist who uses the service
    The correct answer is C.

    This issue is addressed in Standard 9.09(c) of the Ethics Code. According to the Code, "psychologists retain responsibility for the appropriate application, interpretation, and use of assessment instruments, whether they score and interpret such tests themselves or use automated or other services."

    Answer A: Psychologists are responsible for evaluating the validity of automated scoring results.

    Answer B: The psychologist who is using automated scoring tools maintains responsibility for evaluating the results.

    Answer D: The psychologist who is using the scoring service is responsible.
  84. During his first therapy session, an Asian American client tells his non-Asian therapist that he would prefer to see an Asian psychologist. The therapist's best course of action would be to _________________________.

    A.discuss the issue with the client and make a referral if, after the discussion, the client says he still wants to see an Asian therapist

    B.tell the client that she's had experience working with members of minority groups and suggest that they see each other for at least two or three more sessions

    C.realize that the client's preference is a manifestation of resistance and tell him that she thinks it would be best if they saw each other for a few sessions before a referral is made

    D.tell the client that it would be discriminatory, and therefore unethical, for her to make a referral to another therapist on the basis of race alone
    The correct answer is A.

    This question is fairly straightforward. It is never ethical to coerce a client into remaining in therapy. The client will not benefit from treatment if he is not comfortable with the therapist, and it would be unethical to try to "talk him into" additional sessions.

    Answer B: The better approach is to address the client's concerns during the first session and either refer the client or make a plan regarding how to proceed based on the client's wishes.

    Answer C: It would be unethical to pressure the client to continue working with the therapist.

    Answer D: This answer is incorrect, as the client has a right to choose to work with a therapist or not.
  85. When conducting group therapy, you should remind group members ____________________________.

    A.that they are legally bound to maintain confidentiality

    B.that, since they are not legally bound to maintain confidentiality, they should not expect other members to do so

    C.that, as members of a therapy group, they are ethically required to maintain confidentiality

    D.of the importance of maintaining the confidentiality of statements made during group sessions
    The correct answer is D.

    This is an important issue in therapy but poses special problems in multi-client situations. Of the responses given, this is the best approach and is most consistent with Standard 10.03 of the Ethics Code.

    Answer A: Legally, privilege is waived in many states if a third party (e.g., another group member) is present at the time the information is revealed. Also, other group members are not governed by privilege laws. Consequently, there is no legal requirement for confidentiality by group members.

    Answer B: This would NOT be a particularly useful approach in helping to ensure that clients do maintain the confidentiality of other group members.

    Answer C: Group members are not bound by professional codes regarding confidentiality.
  86. An employer interested in increasing the creativity of workers would be best advised to do all of the following except:

    A.provide supportive supervision

    B.let workers know that their work will be critically evaluated

    C.hire employees who score high on a measure of openness to experience

    D.increase the physical distance between workers in the workplace
    The correct answer is B.

    Research investigating the impact of various personal and contextual characteristics on the creativity of employees was recently reviewed by C. Shalley, J. Zhou, and G. Oldham [The effects of personal and contextual effects on creativity: Where should we go from here?, Journal of Management, 30(6), 933-958, 2004]. Whereas the expectation that one's work will be evaluated in a nonjudgmental way (i.e., that evaluations will be developmental rather than judgmental) has a positive impact on creativity, the expectation that one's work will be critically evaluated tends to diminish creativity.

    Answer A: Research has confirmed that supportive supervision has a positive effect on employee creativity.

    Answer C: Of the five personality factors, openness to experience has been found to be most strongly related to creativity.

    Answer D: Reducing the overall density of the work environment has been linked to improvements in worker creativity.
  87. According to Fiedler's contingency model, which of the following is most important for maximizing a supervisor's ability to effectively lead his or her employees?

    A.Each employee is highly motivated.

    B.Each employee is very proficient at his/her job.

    C.The leader has good relationships with his/her employees.

    D.The leader is highly experienced.
    The correct answer is C.

    Fiedler's contingency model identifies three characteristics of the work situation that influence a leader's ability to lead: leader-member relations, task structure, and the leader's position power. Of the three situational characteristics, Fiedler considered leader-member relations to be most critical. According to Fiedler, factors that affect the quality of the relationship between the leader and members are the degree to which the leader perceives members to be similar to him/her and trusts and respects them and the degree to which members like, trust, and respect the leader and are loyal to him/her.

    Answer A: High employee motivation is not a factor in Fiedler's contingency model.

    Answer B: Employee proficiency is not the most important factor for maximizing a supervisor's ability to effectively lead his or her employees.

    Answer D: Highly experienced leaders are not the most important factor for maximizing a supervisor's ability to effectively lead his or her employees.
  88. A psychologist is hired to develop a selection test that will be used to assist in the hiring of management trainees. The psychologist's first step in developing the selection test will be conducting a:

    A.needs assessment

    B.job analysis

    C.job evaluation

    D.formative evaluation
    The correct answer is B.

    For the exam, you want to be familiar with all of the procedures listed in the answers to this question. Additional information about them is provided in the Industrial-Organizational Psychology chapter of the written study materials. To develop a test that will be used to assist in the selection of job applicants, it would be necessary to know the duties and skills needed to successfully perform the job. Therefore, the first step in developing the test will be a job analysis.

    Answer A: A needs assessment (needs analysis) is a first step in designing training programs.

    Answer C: A job evaluation is conducted specifically to determine the relative worth of jobs in order to set salaries and wages.

    Answer D: A formative evaluation is conducted while a training program is being developed to determine if any modifications to the program are required.
  89. Hollander's (1960) concept of "idiosyncrasy credits" is useful for understanding why:

    A.a person exerts less effort when working as a group member than when working alone

    B.a person acts in a way that is opposite of what is requested or expected

    C.a group member is allowed to deviate from group norms without receiving disapproval from other group members

    D.a group member adopts the group decision even when he/she disagrees with it
    The correct answer is C.

    Even if you're unfamiliar with Hollander's (1960) concept, the word "idiosyncrasy" may have helped you identify the correct answer to this question. According to Hollander, group members are allowed to deviate from group norms without negative consequences in certain circumstances - for example, when the member has a history of conformity or is serving as the leader of the group.

    Answer A: This answer describes "social loafing."

    Answer B: This answer describes "psychological reactance."

    Answer D: This answer is not relevant to idiosyncrasy credits.
  90. In terms of initiating structure and consideration, the two core dimensions of leadership, research in organizations suggests that:

    A.consistent with gender stereotypes, female leaders are higher in consideration, while male leaders are higher in initiating structure

    B.contrary to gender stereotypes, female leaders are higher in initiating structure, while males are higher in consideration

    C.contrary to gender stereotypes, males and females do not differ substantially in terms of consideration or initiating structure

    D.contrary to gender stereotypes, males are higher than females in both consideration and initiating structure
    The correct answer is C.

    A considerable body of research has addressed the core dimensions of leadership and, more recently, gender differences on these dimensions. Although male and female leaders differ in terms of decision-making style, overall, their leadership styles do not show consistent discrepancies.
  91. A college student receives a high score on the realistic scale of Holland's Self-Directed Search but a low score on all other scales. This is an example of which of the following?

    A.Congruence

    B.Vocational identity

    C.Differentiation

    D.Individuation
    The correct answer is C.

    Holland's theory of career decision-making incorporates several constructs including congruence, identity, and differentiation. Differentiation refers to the distinctiveness of the individual's profile on the Self-Directed Search. An individual's profile is highly differentiated when he/she obtains a high score on one scale and low scores on all other scales.

    Answer A: As defined by Holland, congruence refers to the degree of consistency between an individual's expressed and assessed interests.

    Answer B: Vocational identity refers to the clarity and stability of the individual's goals and self-perceptions.

    Answer D: Individuation is not one of Holland's constructs.
  92. Studies investigating the relationship between job performance and job satisfaction have most consistently found that:

    A.correlations between job satisfaction and performance are positive, and coefficients are usually moderate to high

    B.correlations between job satisfaction and performance are negative, although negative coefficients are usually low

    C.correlations between job satisfaction and performance are positive, although coefficients are usually low

    D.correlations between job satisfaction and performance are negative, and negative coefficients are usually quite high
    The correct answer is C.

    Contrary to what might be expected, a strong relationship between job satisfaction and performance has not been supported by the research. Correlations between job satisfaction and performance are usually positive but small. Vroom (1964), for example, reported an average correlation coefficient of .14.

    Answer A: While correlations between job satisfaction and performance are usually positive, coefficients are not usually large.

    Answer B: Correlations between job satisfaction and performance are usually positive, not negative.

    Answer D: Correlations between job satisfaction and performance are usually positive, not negative.
  93. Nadler's (1988) systems model of planned change proposes that, to successfully implement change, managers must attend to four factors. One of these is informal organizational elements which consist of:

    A.external (environmental) driving and restraining forces

    B.the employees' knowledge, skills, and expectations

    C.the organization's implicit beliefs, values, and behaviors

    D.the organization's structures, processes, and methods
    The correct answer is C.

    The four factors identified by Nadler (1988) are informal organizational elements, formal organizational elements, individuals (employees and managers), and tasks. The informal organizational elements include implicit beliefs, values, and behaviors (e.g., communication patterns, leadership approach, and work-related norms).

    Answer A: This sounds more like the decision-making technique, force field analysis.

    Answer B: This sounds more like some of the characteristics of Vroom's Expectancy Theory.

    Answer D: This sounds more like Nadler's (1988) formal organizational elements.
  94. In the context of training, "identical elements" refers to the match between:

    A.the expectations of trainees about the potential benefits of a training program and its actual benefits

    B.the information and skills addressed in the training program and the information and skills required by the job

    C.the goals of the training program and the personal goals of the participants in the program

    D.the goals of the training program and the program outcomes desired by management
    The correct answer is B.

    The concept of "identical elements" was introduced by Thorndike and Woodworth (1901). The principle of identical elements proposes that the more similar the training and performance situations are with regard to the presented stimuli and required responses, the greater the transfer of training. Similarity includes both physical similarity (e.g., the similarity of the tasks and environment) and psychological similarity (i.e., the degree to which trainees attach similar meanings to the training and performance situations).

    Answer A: These concepts seem somewhat related to reaction criteria.

    Answer C: This does not refer to "identical elements".

    Answer D: This is somewhat related to "identical elements" but in a very loose way and is therefore not the best answer.
  95. Which of the following is the most commonly used criterion-measure in organizational settings?

    A.Supervisor ratings

    B.Standardized performance tests

    C.Quantitative measures of production

    D.Assessment centers
    The correct answer is A.

    Although some jobs lend themselves to direct (objective) criterion measures, most require that subjective measures be used. Supervisor ratings continue to be the most frequently used method for assessing employees, probably because they are the easiest and quickest way of evaluating employees.

    Answer B: Performance tests are more likely to be used as predictors than criteria.

    Answer C: Quantitative indices (e.g., number of units produced) are unavailable for many jobs.

    Answer D: Assessment centers are costly in terms of time and money and are used in large organizations for the purpose of managerial placements and promotions.
  96. According to Dawis, England, and Lofquist's (1964) theory of work adjustment, the probability that a person will be forced out of the work environment is inversely related to his/her:

    A.satisfaction

    B.satisfactoriness

    C.job tenure

    D.organizational commitment
    The correct answer is B.

    Satisfactoriness is the extent to which an employee is able to perform a job, and there is an inverse relationship between how well an employee performs a job and the probability that the worker will be forced out of the work environment (fired).

    Answer A: In the theory of work adjustment, satisfaction refers to the degree to which the characteristics of the job corresponds to the needs and values of the worker. According to this theory, the probability that a person will voluntarily leave the work environment is inversely related to the worker's satisfaction.

    Answer C: The theory of work adjustment predicts that job tenure is a joint function of satisfaction and satisfactoriness.

    Answer D: Organizational commitment refers to the strength of an employee's psychological attachment to an organization and is not part of the theory of work adjustment model.
  97. When using a Behaviorally-Anchored Rating Scale (BARS), scale points are "anchored" with:

    A.empirically-derived weights

    B.job dimensions

    C.criterion outcomes

    D.critical incidents
    The correct answer is D.

    In a Behaviorally-Anchored Rating Scale (BARS), the points on the scale are "anchored" with descriptions of specific job behaviors. Another name for behavioral anchors is critical incidents.
  98. When a rater's ratings on a criterion measure are biased by his or her knowledge of the ratee's performance on the predictor, this is referred to as:

    A.criterion irrelevance

    B.criterion unreliability

    C.criterion contamination

    D.criterion deficiency
    The correct answer is C.

    Criterion contamination occurs when a criterion measure is "contaminated" by an extraneous variable. Contamination often occurs when a rater's knowledge of a ratee's predictor performance biases ratings on the criterion measure.

    Answer A: Relevance refers to the measure's significance in terms of the ultimate criterion.

    Answer B: A criterion measure is unreliable when scores are affected to a large degree by measurement error.

    Answer D: A criterion is deficient when it does not measure important aspects of the ultimate criterion.
  99. In addition to identifying "bounded rationality" as an impediment to rational decision-making, Herbert Simon is known for his work on:

    A.artificial intelligence

    B.job burnout

    C.groupthink

    D.organizational culture
    The correct answer is A.

    Although Herbert Simon is probably best known for his work on individual decision-making, he was also a pioneer in the field of artificial intelligence. Simon believed there are two main goals in using and studying computers: (1) to augment human intelligence and (2) to help understand how humans think.
  100. As defined in the EEOC Uniform Guidelines, "unfairness" is a problem when:

    A.different groups obtain consistently different scores on the predictor but similar scores on the criterion

    B.different groups obtain consistently different scores on the criterion but similar scores on the predictor

    C.the predictor has significantly different validity coefficients for different groups

    D.the predictor is equally invalid for all groups
    The correct answer is A.

    Unfairness is one of the possible causes of adverse impact. A test is "unfair" when it has a similar validity coefficient for members of two (or more) groups and members of the groups have similar criterion performance, but members of one group consistently score lower on the predictor than members of the other group(s). In this situation, if the same predictor cutoff is used for all individuals, there may be an adverse impact for members of the low-scoring group.
  101. According to Herzberg's "two-factor theory," a disgruntled blue-collar worker is most likely to say he is satisfied with his job if he is:

    A.given a raise

    B.given fewer tasks to do

    C.put on a piece-rate pay system

    D.given more responsibility
    The correct answer is D.

    Herzberg's two-factor theory divides job components into two types: hygiene factors and motivator factors. According to this theory, satisfaction and motivation are increased by providing an employee with motivator factors; i.e., with greater responsibility, autonomy, challenge, etc.

    Answer A: For Herzberg, money is a hygiene factor and, consequently, can produce dissatisfaction when it is perceived to be inadequate but does not contribute to satisfaction or motivation, even when it is perceived to be adequate.

    Answer B: From Herzberg's perspective, being given fewer tasks to do would not lead to greater satisfaction.

    Answer C: Being put on a piece-rate pay system is not consistent with two-factor theory.
  102. Overtraining is most associated with:

    A.peak performance

    B.enhanced transfer of training

    C.a drug-like "high"

    D.decreased motivation and performance
    The correct answer is D.

    Note that this question is asking about overtraining, not overlearning. The term overtraining appears in the sports psychology literature; overlearning is used in the learning and training literature. In sports psychology literature, overtraining (staleness) refers to excessive training and the resulting physical and psychological symptoms. The undesirable effects of overtraining include premature fatigue during exercise, reduced performance, emotional instability, decreased motivation, and increased susceptibility to infection.

    Answer A: Peak performance would be the best answer if the question were asking about overlearning.

    Answer B: Enhanced transfer of training also better fits overlearning.

    Answer C: A drug-like "high" is not associated with overtraining or overlearning.
  103. Research on women working outside the homes suggests that combining work and family roles is most associated with:

    A.stress-related illness

    B.reduced involvement in family roles

    C.enhanced self-esteem

    D.low job commitment
    The correct answer is C.

    This topic was addressed in a review of the research on women and work (S. D. Phillips and A. R. Imhoff, Women and career development: A decade of research, Annual Review of Psychology, 48, 31-59, 1997). Empirical research suggests that, overall, dual roles for women have positive effects on self-esteem and other aspects of well-being.
  104. In the context of expectancy theory, valence refers to:

    A.degree of effort

    B.level of performance

    C.need strength

    D.subjective worth of outcomes
    The correct answer is D.

    Expectancy theory distinguishes between three types of beliefs that contribute to motivation - expectancy, instrumentality, and valence. In the context of expectancy theory, valence refers to the value of available outcomes to the individual.
  105. The Taylor-Russell tables will indicate that a new selection test is most likely to be helpful in choosing job applicants who will be satisfactory employees when:

    A.the proportion of current employees who are considered satisfactory is less than 25%

    B.there are only a few job openings and many applicants for the job

    C.the majority of applicants have jobs skills in the moderate (average) range

    D.the test has adequate "differential validity"
    The correct answer is B.

    Use of the Taylor-Russell tables estimates the percent of employees who will be satisfactory if a new selection technique is used and requires three pieces of information - the test's validity coefficient, the selection ratio, and the base rate. All other things being equal, a new selection test will be most useful for selecting job applicants when the selection ratio is low (i.e., there are few jobs and many applicants).

    Answer A: The proportion of current employees who are considered successful is the "base rate." A new selection test will be most useful when the base rate is moderate (around 50%).

    Answer C: The fact that "the majority of applicants have jobs skills in the moderate (average) range" would not be a factor that is considered when using the Taylor-Russell tables to evaluate the usefulness of a selection test.

    Answer D: "Differential validity" occurs when a measure is valid for one group but is not valid for another group and is not a factor that is considered when using the Taylor-Russell tables.
  106. Research on Total Quality Management suggests that, when it fails, this is often because:

    A.the manager didn't adapt his/her style to the demands of the situation

    B.the employees were not given adequate training

    C.the employees were not sufficiently involved in problem-solving and decision-making

    D.the manager often had to "satisfice" rather than "optimize" because of limited resources
    The correct answer is C.

    Knowing that Total Quality Management (TQM) emphasizes employee involvement would have helped you choose the correct response to this question. TQM is a theory of management that emphasizes customer satisfaction, employee involvement, and continuous change. Studies looking at TQM failures have found that, despite the goal of commitment to employee involvement, employees often do not participate fully in problem-solving and decision-making.
  107. A low selection ratio and a moderate base rate are useful for:

    A.maximizing incremental validity

    B.reducing the likelihood of adverse impact

    C.increasing discriminant validity

    D.reducing measurement error
    The correct answer is A.

    For the exam, you want to have the terms selection ratio and base rate linked to the concept of incremental validity. A predictor is most likely to increase decision-making accuracy (i.e., have good incremental validity) when there are many applicants to choose from (a low selection ratio) and when the current technique produces a moderate number of correct decisions (moderate base rate).

    Answer B: Selection ratio and base rate are not directly related to adverse impact.

    Answer C: Discriminant validity refers to low correlations between measures that assess different traits. It is used as evidence of construct validity.

    Answer D: Selection ratio and base rate are not relevant to the extent of measurement error.
  108. According to Maslow's need hierarchy theory, once a person's physiological needs have been satisfied, that person will be motivated by his/her __________ needs.

    A.social

    B.safety

    C.existence

    D.esteem
    The correct answer is B.

    Maslow's need hierarchy theory predicts that humans have five basic needs that emerge in a hierarchical order. The five needs, in order, are physiological, safety, social, esteem, and self-actualization. Need hierarchy theory predicts that physiological needs are the first needs to act as motivators; once they are satisfied, safety needs become the source of motivation; and so on.

    Answer A: Social needs are the focus after physiological and safety needs are met.

    Answer B: Existence is one of the needs in Alderfer's ERG Theory.

    Answer D: Esteem needs are not motivators until physiological, safety, and social needs have been satisfied.
  109. A colleague of yours, an industrial psychologist, designs a study to investigate the relationship between job satisfaction and mental and physical health. Based on your knowledge of the research in this area, you tell him that:

    A.there is no consistent relationship between job satisfaction and mental or physical health

    B.there is a positive relationship between job satisfaction and mental and physical health

    C.there is a positive relationship between job satisfaction and mental health but no relationship between job satisfaction and physical health

    D.there is a positive relationship between job satisfaction and physical health but no relationship between job satisfaction and mental health
    The correct answer is B.

    Even if you are unfamiliar with the research in this area, it seems logical that job dissatisfaction might be either a cause or effect of poor physical and/or mental health. Several studies have found a positive relationship between job satisfaction and both mental and physical health. Note, however, that the research has not demonstrated a causal relationship between job satisfaction and mental/physical health or vice-versa, only that they are correlated.
  110. Which of the following refers to the performance of a task in which the meaning of a stimulus depends on the stimuli it is paired with?

    A.concept learning

    B.higher-order conditioning

    C.configural learning

    D.incidental learning
    The correct answer is C.

    Configural learning refers to responding to two or more stimuli based on their combination rather than on the individual experience of any of those stimuli alone.

    Answer A: Concept learning refers to learning a task in which one classifies objects by being shown example(s) of the object along with their classes or categories. Then the learner will generalize what they have learned to a similar object.

    Answer B: Higher-order conditioning refers to a situation in which a previously neutral stimulus is paired with a conditioned stimulus to produce the same conditioned response as the conditioned stimulus.

    Answer D: Incidental learning is learning that is not premeditated, deliberate, or intentional and that is acquired as a result of some other, possibly unrelated, mental activity.
  111. Which of the following is based on the notion that initiation of one item stored in memory travels through associated links to stimulate another item?

    A.Response deprivation theory

    B.Response-outcome expectation

    C.Sutherland-Mackintosh attentional theory

    D.Spreading activation theory
    The correct answer is D.

    Spreading activation theory is the view that semantic memory is organized in terms of associations between concepts and properties of concepts.

    Answer A: The response deprivation theory is the idea that when a contingency restricts access to an activity, it causes that activity to become a reinforcement.

    Answer B: Response-outcome expectation is the belief that a particular response leads to a specific consequence or outcome.

    Answer C: The Sutherland-Makintosh attentional theory is the theory that attention to the relevant dimension is strengthened in the first stage, and association of a particular response to the relevant stimulus occurs in the second stage of discrimination learning.
  112. Which of the following explains associative learning in which simultaneous firing of cells leads to pronounced increases in synaptic strength between those cells, and provides a biological basis for the pairing of stimulus and response in classical conditioning? 

    A.Overexpectation effect

    B.Reactive inhibition

    C.Hebbian rule

    D.Partial reinforcement effect
    The correct answer is C.

    The Hebbian rule is a learning rule that describes how neuronal activities influence the connection between neurons (i.e., synaptic plasticity).

    Answer A: The over expectation effect refers to the decrease in the conditioned response that occurs when two separately conditioned stimuli are combined into a compound stimulus for further pairings with the unconditioned stimulus. 

    Answer B: Reactive inhibition is the temporary suppression of behaviors due to the persistence of a drive state after unsuccessful behavior.

    Answer D: The partial reinforcement effect, also known as intermittent reinforcement, occurs when a response is reinforced intermittently. With this type of reinforcement learned behaviors are acquired slower, but responses are more resistant to extinction.
  113. _________ is the process where large pieces of information are divided into smaller units to retain it in short-term memory.

    A.Rote rehearsal

    B.Elaboration

    C.Imagery

    D.Chunking
    The correct answer is D.

    Chunking is the grouping of related items of information to retain to short-term memory. 

    Answer A: Rote rehearsal refers to saying material repeatedly as a method of committing it to mind.

    Answer B: Elaboration is the process of interpreting or embellishing information to be remembered or of relating it to other material already known and in memory.

    Answer C: Imagery is the formation of mental pictures to enhance the processing of information into memory.
  114. Using clicker training is an example of _______ and has been extensively studied in animals. An illustration would include a horse persistently watching, approaching, touching, and even mouthing the clicker before shifting its attention and moving to the location where food is delivered.

    A.avoidance conditioning

    B.instinctive drift

    C.sign tracking

    D.noncontingent reinforcement
    The correct answer is C.

    Sign tracking, in conditioning, refers to elicited behavior directed towards a stimulus that is reliably paired with a primary reinforcer.

    Answer A: Avoidance conditioning is the establishment of behavior that prevents or postpones aversive stimulation.

    Answer B: Instinctive drift is the tendency of learned behavior to gradually return to a more innate behavior. For example, raccoons trained to drop coins into a container will eventually begin to dip the coins into the container, pull them back out, rub them together, and dip them in again. The learned behavior of dropping coins becomes more representative of the innate behavior of food washing.

    Answer D: Noncontingent reinforcement is the use of positive reinforcement that is not related to the occurrence of a target behavior. It involves delivering reinforcement on a fixed-time schedule independent of whether the individual exhibits the target behavior during the interval.
  115. Which of the following type of memory occurs in the occipital lobe and does not last longer than a half-second after the perception of the object?

    A.Procedural memory

    B.Haptic memory

    C.Echoic memory

    D.Iconic memory
    The correct answer is D.

    Iconic memory refers to short-term visual memories that form in the occipital lobe.

    Answer A: Procedural memory refers to a skilled memory or the memory of a highly practiced behavior.

    Answer B: Haptic memory is sensory memory for a tactile experience that is stored in the parietal lobe.

    Answer C: Echoic memory is sensory memory that registers specific to auditory information (i.e., sounds) and is stored in the primary auditory cortex.
  116. Which of the following is a technique, in which undesirable behavior is weakened and its occurrence decreases, typically by moving the individual away from the area that is reinforcing the behavior?

    A.Time-out

    B.Negative reinforcement

    C.Response cost

    D.Positive punishment
    The correct answer is A.



    Time-out is a behavior modification technique that is typically considered a form of negative punishment.  Time-out involves removing all sources of positive reinforcement for a brief, prespecified period of time following a behavior to decrease the behavior. 

    Answer B: Negative reinforcement involves removing a stimulus after a behavior occurs in order to increase the behavior. Negative reinforcement strengthens, not weakens, the behavior. 

    Answer C: Response cost is another application of negative punishment and is most commonly associated with token economies. It involves removing a specific reinforcer each time the target behavior is performed. Answer D: Positive punishment involves presenting an unfavorable outcome or event following an undesirable behavior to decrease the behavior. For example, positive punishment may be grounding a child after the child throws a tantrum.
  117. In a classroom, a student yells out an answer and is reprimanded by the teacher every other time. However, when the student raises her hand, the teacher calls on her every six times. Assuming attention is the reinforcer, yelling will occur more often than hand-raising based on which of the following?

    A.Ainslie-Rachlin Model

    B.Matching law

    C.Trans-situationality

    D.Delayed gratification paradigm
    The correct answer is B.

    The matching law suggests that when different schedules of reinforcement are available at the same time for different behaviors, individuals will allocate their behavior according to the relative rates of reinforcement available for each option.

    Answer A: Ainslie-Rachlin model is a theory of self-control choices that explains why an individual's preferences can shift from a larger, delayed reinforcer to a smaller, more immediate reinforcer as the time of delivery approaches.

    Answer C: Trans-situationality is the theory that once a stimulus is determined to be a reinforcer in one situation, it will also serve as a reinforcer in other situations.

    Answer D: The delayed gratification paradigm is the act of resisting an impulse to take an immediately available reward in the hope of obtaining a more valued reward in the future. The ability to delay gratification is essential to self-regulation or self-control.
  118. In classical conditioning, ________ occurs when individuals are unable to form an association between a new neutral stimulus and an unconditioned stimulus due to interference by the neutral stimulus.

    A.overshadowing

    B.blocking

    C.experimental neurosis

    D.reciprocal inhibition
    The correct answer is B.

    Blocking occurs when an association between a conditioned stimulus (CS) and an unconditioned stimulus (US) has been made and, subsequently, the presence of the CS blocks a developing association between a new neutral stimulus and the US.

    Answer A: Overshadowing occurs when two or more stimuli are present, and one stimulus produces a stronger response than the other because it is more salient.

    Answer C: Experimental neurosis is an abnormal behavioral condition produced in a laboratory setting. The subject is typically placed in a problem-solving or discernment scenario they cannot solve because it is too difficult or impossible, which can result in erratic or altered behavior.

    Answer D: Reciprocal inhibition is a technique in behavior therapy that aims to replace an undesired response (e.g., anxiety) with the desired one by counterconditioning.
  119. Reinforcement delivered on a set schedule versus a positive response is known as: 

    A.Differential reinforcement of incompatible behavior (DRI)

    B.Differential reinforcement of alternative behavior (DRA)

    C.Noncontingent reinforcement

    D.Differential reinforcement of other behavior (DRO)
    The correct answer is C.

    Noncontingent reinforcement is the presentation of a reinforcer, independent of the presence of a specific behavior.

    Answer A: DRI entails only reinforcing behaviors that are incompatible with the problem behavior while withholding reinforcement for the problem behavior. Specifically, only behaviors that cannot occur simultaneously with the problem behavior are reinforced.

    Answer B: DRA entails reinforcing a behavior that serves as a viable alternative for the problem behavior but is not necessarily incompatible with the problem behavior.

    Answer D: DRO entails delivering reinforcement whenever the problem behavior does not occur during a predetermined amount of time.
  120. The concept of _______ refers to the idea that some stimuli, despite repeated pairings, do not become associated with a specific unconditioned stimulus.

    A.prepared learning

    B.contrapreparedness

    C.counterconditioning

    D.overshadowing
    The correct answer is B.

    Contrapreparedness is an inability to associate a specific conditioned stimulus and a specific unconditioned stimulus despite repeated conditioning experiences.

    Answer A: Prepared learning refers to the ability adaptive for evolution, allowing certain associations to be learned more readily than others.

    Answer C: Counterconditioning is the elimination of a conditioned response when the conditioned stimulus is paired with an opponent unconditioned stimulus.

    Answer D: Overshadowing, in a compound conditioning situation, refers to the prevention of conditioning to one stimulus due to the presence of a more salient or intense stimulus.
  121. Dopamine is central to reinforcing the quality of events and is found in which of the following areas of the limbic system that influences behavior reinforcement?

    A.Basal ganglia

    B.Amygdala

    C.Hippocampus

    D.Nucleus accumbens
    The correct answer is D.

    The nucleus accumbens is a brain structure in the limbic system that plays a significant role in the influence of reinforcement on behavior where dopamine is central to the reinforcing quality of events.

    Answer A: The basal ganglia are involved in the storage and retrieval of procedural memories.

    Answer B: The amygdala has been implicated in fear conditioning and sends projections to the hypothalamus and brainstem nuclei that mediate various fear responses.

    Answer C: The hippocampus encodes new episodic memories in humans. It also plays a role in the capacity to reduce the associability of conditioned responses.
  122. Which of the following involves presenting and withdrawing the conditioned stimulus (CS) and unconditioned stimulus (US) at the same time?

    A.Simultaneous conditioning

    B.Backward conditioning

    C.Trace conditioning

    D.Delay conditioning
    The correct answer is A.



    • Pavlov found that the order and timing of the presentation of the CS and US were important determinants of the success of classical conditioning, and he distinguished between the following temporal relationships: Delay, trace, simultaneous, and backward conditioning.  Simultaneous conditioning is even less effective than trace conditioning and involves presenting and withdrawing the CS and US at the same time.
    • Answer B: Backward conditioning entails presenting the US prior to the CS.

    Answer C: Trace conditioning entails presenting and terminating the CS prior to presenting the US.

    Answer D: Delay conditioning involves presenting the CS so that it precedes and overlaps presentation of the US.
  123. A rat is trained to press a lever to receive food. During this, the rat is occasionally exposed to a series of brief shocks that are preceded by a tone. As a result, when the rat hears the tone, lever pressing is reduced. This is an example of which of the following?

    A.Conditioned suppression

    B.Attribute conditioning

    C.Automatic reinforcement

    D.Long-delay conditioning
    The correct answer is A.

    Conditioned suppression is a procedure in which the conditioned stimulus signals that an aversive event is coming. The measure of conditioning is the suppression of ongoing behavior when the conditioned stimulus is presented.

    Answer B: Attribute conditioning occurs when the attributes or characteristics of one stimulus are transferred to another stimulus.

    Answer C: Automatic reinforcement refers to behavior that is derived from sensory stimulation that occurs as a result of performing the behavior.

    Answer D: Long-delay conditioning is a type of classical conditioning in which the onset of the conditioned stimulus precedes that of the unconditioned stimulus by at least several seconds and continues until the unconditioned stimulus is presented.
  124. In the first months of life, infants can locate objects through _______, which involves learning the association between bodily reaction and a particular position in space.

    A.avoidance learning

    B.place learning

    C.response learning

    D.latent learning
    The correct answer is C.

    Response learning refers to performing a specific series of movements or responses (e.g., eye movement, or reach).

    Answer A: Avoidance learning is the process by which an individual learns a behavior or response to avoid a stressful or unpleasant situation.

    Answer B: Place learning is the learning of locations or physical positions of goals (e.g., where food can be found).

    Answer D: Latent learning occurs without any obvious reinforcement of the behavior.
  125. According to Beck (1976), overgeneralization, personalization, and magnification are examples of which of the following?

    A.Cognitive triad

    B.Cognitive distortions

    C.Cognitive schema

    D.Automatic thoughts
    The correct answer is B.

    Cognitive distortions are faulty or unhelpful ways of thinking. Overgeneralization, personalization and blaming, magnification, all-or-nothing thinking, and discounting the positive are some examples.

    Answer A: The cognitive triad was developed by Beck and describes how depressed adults tend to think about the world (i.e., negative views). Specifically, the triad refers to thoughts about self, world, and future.

    Answer C: Cognitive schema is described in Piaget's theory of cognitive development. Schemas are the building blocks for knowledge acquisition.

    Answer D: Automatic thoughts are instantaneous, habitual, and nonconscious. Automatic thoughts affect an individual's mood and actions.
  126. Food deprivation is a _______. If an individual is hungry, food is reinforcing, but if an individual is satiated, food is less reinforcing.

    A.stimulus control

    B.motivating operation

    C.unlearned reinforcer

    D.unlearned aversive stimuli
    The correct answer is B.

    Motivating operations are motivations that encourage or discourage certain behaviors. Their purpose is to enhance or reduce the reinforcement value.

    Answer A: Stimulus control describes situations in which a behavior is triggered by the presence or absence of some stimulus.

    Answer C: Unlearned reinforcer is a stimulus that is a reinforcer, though not as a result of pairing with another reinforcer. Examples include food, water, and sex.

    Answer D: Unlearned aversive stimuli is a stimulus that is aversive though not as the result of pairing with other aversive stimuli. An example is an electric shock.
  127. Treisman and Gelade's (1980) feature-integration theory (FIT) would be of most interest to a psychologist researching which of the following?

    A.Arousal

    B.Memory consolidation

    C.Visual attention

    D.Executive functioning
    The correct answer is C.

    Feature-integration theory is a two-stage theory of visual attention that explains how an individual combines pieces of observable information about an object to form a complete perception of the object.

    Answer A: For research on arousal, a psychologist may be interested in the Yerkes-Dodson Law which suggests that arousal levels can affect an individual's performance abilities.

    Answer B: For research on memory consolidation, a psychologist may be interested in multiple trace theory which posits that each time some information is presented to an individual, it is neurally encoded in a unique memory trace composed of a combination of its attributes.

    Answer D: Executive functioning is an umbrella term used to encompass cognitive processes, including planning, working memory, attention, inhibition, self-monitoring, self-regulation, and initiation.
  128. Getting stuck in a rut, repeating known errors, or using an injured arm or leg out of habit are all examples of which of the following?

    A.Latent inhibition

    B.Blocking

    C.Overshadowing

    D.Pseudoconditioning
    The correct answer is A.

    Latent inhibition suggests that it is often easier to learn something new than to unlearn something familiar.

    Answer B: Blocking refers to the finding that less is learned about the relationship between a stimulus and an outcome if pairings are conducted in the presence of a second stimulus that has previously been established as a reliable predictor of that outcome.

    Answer C: Overshadowing occurs when two or more stimuli are present, and one stimulus produces a stronger response than the other due to relevance or salience.

    Answer D: Pseudoconditioning refers to the elicitation of a response by a previously neutral stimulus when it is presented following a series of occurrences of a conditioned stimulus.
  129. Which of the following determines an individuals' perceptions and behaviors based on the causes individuals give for interpersonal and performance situations?

    A.Stress innoculation

    B.Attribution retraining

    C.Self-control therapy

    D.Problem-solving therapy
    The correct answer is B.

    Attribution retraining focuses on altering the individual's perceptions of the causes of his or her problematic behavior to help clients attribute their failures to external, unstable, and specific factors and successes to internal, stable, and global factors. 

    Answer A: Stress inoculation involves teaching coping skills to manage stress and anxiety. Techniques include training in deep muscle relaxation, cognitive restructuring, breathing exercises, thought stopping, role-playing, and guided self-dialogue.

    Answer C: Self-control therapy is a brief form of therapy that is usually conducted as group therapy. It involves didactic presentations, instructional exercises to teach concepts and skills, and the application of these skills to the day-to-day lives or participants through homework assignments. This therapy type has been applied to the treatment of depression in children, adults, and geriatric populations.
  130. A tone, light, buzzer, or any stimulus that precedes a shock becomes which of the following in conditioning training?

    A.Motivating operations

    B.Unlearned aversive stimulus

    C.Conditioned aversive stimulus

    D.Unlearned reinforcer
    The correct answer is C.

    A conditioned aversive stimulus is an initially neutral stimulus that becomes aversive after repeated pairing with an unconditioned aversive stimulus.

    Answer A: Motivating operations are the motivations that encourage or discourage certain behaviors with the purpose to enhance or reduce the reinforcement value.

    Answer B: An unlearned aversive stimulus is a stimulus that is aversive though not from pairing with other aversive stimuli.

    Answer D: An unlearned reinforcer is a stimulus that is a reinforcer, though not as a result of pairing with another reinforcer.
  131. In an experiment, a tone and light are repeatedly presented together to a hungry dog. Even though the tone has never been paired with food, the dog salivates when the tone is presented. This is an example of which of the following?

    A.Conditioned inhibition

    B.Temporal conditioning

    C.Higher-order conditioning

    D.Sensory preconditioning
    The correct answer is D.

    Sensory preconditioning involves initially pairing two neutral stimuli (i.e., A and B) and subsequently pairing A with an unconditioned stimulus. If B eventually elicits a response, then sensory preconditioning has occurred.

    Answer A: Conditioned inhibition is an internal state that has been behaviorally learned and prevents an individual from responding to stimuli that would normally elicit a response. This type of inhibition can be conditioned through the use of punishers (e.g., electric shock) or lack of reinforcers.

    Answer B: Temporal conditioning is a procedure where an unconditioned stimulus is presented at regular intervals but in the absence of an accompanying conditioned stimulus.

    Answer C: Higher-order conditioning is a procedure where a conditioned stimulus (CS) of one experiment acts as the unconditioned stimulus (US) of another to condition a neutral stimulus (NS)
  132. Huntington's disease, Marfan's syndrome, and Von Willebrand's disease are due to:

    A.an autosomal dominant gene

    B.an autosomal recessive gene

    C.an X-linked dominant gene

    D.an X-linked recessive gene
    The correct answer is A.

    All three disorders are caused by an autosomal dominant gene, which means that they occur in the presence of only one gene on a chromosome that is not a sex (X or Y) chromosome.

    Answer B: In an autosomal recessive condition, each parent carries one copy of the mutated gene, but they typically do not show signs and symptoms of the condition.

    Answer C: X-linked disorders are single gene disorders that result from the presence of a mutated gene on the X chromosome. When the gene acts in a dominant manner both males and females can display the trait or disorder when they have only one copy of the gene inherited from a parent.

    Answer D: Because females (XX) have two copies of the X chromosome but males (XY) only have one copy, X-linked recessive disorders are more common in males.
  133. Which of the following best describes Kohlberg's claim about the relationship between moral judgment and moral action?

    A.There is a strong relationship between moral judgment and moral action at all stages of moral development.

    B.The relationship between moral judgment and moral action is strongest at the lower stages of moral development.

    C.The relationship between moral judgment and moral action is strongest at the higher stages of moral development.

    D.There is no predictable relationship between moral judgment and moral action at any stage of moral development.
    The correct answer is C.

    Kohlberg distinguished between moral judgment and moral action, and his theory of moral development focuses on moral judgment. He proposed that there is a greater correspondence between moral judgment and action at the higher stages of moral development because the higher stages employ more stable and generalizable standards.

    Answer A: Kohlberg does not propose equal correspondence between moral judgement and action at all stages.

    Answer B: This answer is opposite of Kohlberg's view.

    Answer D: Kohlberg does predict an expected correspondence between moral judgment and action at the higher stages of moral development.
  134. With regard to language acquisition, a child's use of his/her knowledge of the meaning of words to make inferences about their grammatical type or structure is referred to as:

    A.semantic bootstrapping

    B.prosodic bootstrapping

    C.syntactic bootstrapping

    D.morphological bootstrapping
    The correct answer is A.

    Semantic refers to "meaning," and semantic bootstrapping is a term used in the field of psycholinguistics to describe how children use their knowledge of the meaning of words to infer their syntactical category which, in turn, allows them to understand syntactical rules and construct grammatically correct sentences.

    Answer B: Prosodic bootstrapping refers to using the prosody (pitch, rhythm, etc.) of an utterance to make deductions about syntax.

    Answer C: Syntactic bootstrapping refers to using syntactical information to make deductions about a word's meaning.

    Answer D: Morphological bootstrapping refers to using knowledge about morphemes to deduce the syntax or meaning of a word , e.g., deducing that a word is an action word (verb) because it ends in "ing."
  135. A child who has just progressed into Kohlberg's conventional level of moral development has been caught stealing from other children at school. The child's parents would be best advised to do which of the following to discourage the child's stealing?

    A.Tell her that she will receive a present at the end of each week that she does not steal.

    B.Help her understand how the children she steals from feel about having their things taken.

    C.Tell her that stealing is against the law.

    D.Have her personally return the stolen items to their owners.
    The correct answer is B.

    • The earlier stage in Kohlberg's conventional level of moral development is referred to as the "Good Boy/Good Girl" stage and is characterized by a strong belief in the Golden Rule. Children in this stage are cognitively able to put themselves in the place of others and consider the feelings of others when making moral judgments. Consequently, this approach would be most effective with a child who is in this stage.
    •   
    • Answer A: This approach would probably be most effective for a child in the later stage of preconventional morality. 

    Answer C: Law and authority determine "right" and "wrong" for children likely occur later within the conventional morality stage. This child just progressed into Kohlberg's conventional level of moral development. 

    Answer D: Although this approach may be beneficial, it is not addressed by Kohlberg's theory.
  136. The strength of the relationship between authoritative parenting and adolescent scholastic achievement is moderated most by which of the following variables?

    A.Gender

    B.Ethnicity

    C.Age

    D.Socioeconomic status
    The correct answer is B.

    Steinberg et al. found that an authoritative parenting style is predictive of successful school performance in adolescence. However, while predictive accuracy remains the same regardless of gender, socioeconomic status, or age (younger versus older adolescent), the effects of authoritativeness are moderated by culture/ethnicity. Specifically, the relationship between authoritative parenting and academic achievement is much weaker for African-American adolescents, apparently because they are more strongly influenced by their peers than by their parents. [Steinberg, L. et al. (1992). Authoritative parenting, school involvement, and encouragement to succeed, Child Development, 63, 1266-1281, 1992).]
  137. Alcohol consumption by a pregnant woman is likely to have the most adverse effects on her baby's prenatal development when the woman drinks during:

    A.the first trimester

    B.the second trimester

    C.the third trimester

    D.the first and the third trimester
    The correct answer is A.

    Alcohol consumption by a woman during pregnancy can have detrimental effects on the developing fetus, with the severity and extent of the effects depending on the amount of alcohol consumed and the time during which it is consumed. There does not appear to be any safe time for alcohol consumption during pregnancy. However, the worst effects are associated with drinking during the first trimester.
  138. Research has found that the effects of sexual abuse for a child depend on several factors. Which of the following most accurately describes a finding of these studies?

    A.The effects are less severe when the abuse was committed by a family member.

    B.The effects are less severe when the abuse was committed by a stranger.

    C.The younger the child is at the time of the initial abuse, the less severe the effects.

    D.The younger the age of the perpetrator, the more severe the effects.
    The correct answer is B.

    Although many individuals exhibit long-term adverse effects of childhood sexual abuse, certain conditions have been linked with better outcomes. Abuse by a stranger is associated with less severe effects, as noted in the Lifespan Development chapter.

    Answer A: The effects of abuse are usually worse when the perpetrator is a family member.

    Answer C: Younger age at the time of initial abuse is associated with more damaging outcomes.

    Answer D: The older the age of the perpetrator, the more severe the effects are on the victim.
  139. The ability to achieve native-like pronunciation of a second language is:

    A.more likely when exposure begins in early or middle childhood

    B.more likely when exposure begins in adolescence

    C.more likely when exposure begins in early adulthood

    D.unrelated to the age when exposure begins
    The correct answer is A.

    Individuals exposed to a second language during childhood are more likely to speak with a native accent. In other words, pronunciation is the one aspect of language learning for which there is fairly consistent evidence that "younger is better."

    Answer B: Adolescents may initially make faster progress in acquiring a second language (especially with regard to syntax and morphology), but they are slower in demonstrating a native accent.

    Answer C: Adults, like adolescents, make faster progress in acquiring a second language (especially with regard to syntax and morphology), but they are slower in demonstrating a native accent.

    Answer D: The evidence is that age is an important factor in developing a native accent.
  140. Maternal depression is a risk factor for the emotional, behavioral, and cognitive development of children. For example, there is evidence that toddlers of mothers with severe depression tend to be:

    A.excessively acquiescent

    B.hypervigilant and anxious

    C.antagonistic and belligerent

    D.passively noncompliant
    The correct answer is D.

    Several studies have confirmed that the toddlers of mothers with chronic and severe depressive symptoms are more likely than other children to have adjustment problems. For example, there is evidence that children of severely depressed mothers have low levels of independence and tend to be passively noncompliant (i.e., they ignore requests rather than more actively refuse to comply). [ e.g., Kuczynski, L. & Kochanska, G. (1990). Development of children's noncompliance strategies from toddlerhood to age 5. Developmental Psychology, 26(3), 398-408.]
  141. In a research study, a child sits in front of a three-dimensional model of a mountain scene while another person sits on the opposite side of the mountain scene. The child is then asked to first choose a photograph that depicts the scene that he or she has observed and then to choose the picture that depicts what the other person observed. Based on research on Piaget's stages of cognitive development, you predict that children 3 to 5 years of age will most likely:

    A.pick the correct picture for their own point of view and pick the same (incorrect) picture for the point of view of the other person

    B.pick the correct picture for their own point of view and pick a different (but incorrect) picture for the point of view of the other person

    C.pick the correct pictures for their own point of view and the point of view of the other person

    D.pick incorrect pictures for their own point of view and the point of view of the other person
    The correct answer is A.

    This response is consistent with the results of research conducted by Piaget and others, which found that preoperational children were usually unable to recognize that another person's view of the mountain differed from their own. (Note, however, that some recent research suggests that young children's perspective-taking may be affected by the type of task or other variables.)

    Answers B, C, and D: Based on Piaget's stages of cognitive development and the information presented in the scenario, you can predict that children 3 to 5 years of age will most likely pick the correct picture for their own point of view and pick the same (incorrect) picture for the point of view of the other person.
  142. Current theories of gender development hypothesize:

    A.that cognitive and social factors interact in the development of gender identity

    B.that environment reinforces and influences the development of gender identity

    C.that a conceptual understanding of gender categories precedes the development of gender identity

    D.that children form gender schemas that direct them to actively seek information about gender and this motivates and organizes their behavior
    The correct answer is A.

    Current theories suggest that there is a dynamic interaction between cognitive development and processes of socialization. [See Martin, C. L., & Ruble, D. N. (2010). Patterns of gender development. Annual review of psychology, 61, 353381.]

    Answer B: This represents Mischel's 1966 view of gender development, in which behavior precedes cognition.

    Answer C: This is the gender development theory suggested by Kohlberg, 1966, in which the understanding of gender categories determines gender identification.

    Answer D: Gender schema theory (e.g. Bem, 1981) suggests that the formation of gender schemas precedes gender development.
  143. Which of the following aspects of memory is most negatively impacted by increasing age in adulthood?

    A.Sensory memory

    B.Secondary memory

    C.Memory span

    D.Implicit memory
    The correct answer is B.

    The effects of increasing age vary for different aspects of memory. Secondary memory is the aspect of long-term memory that contains newly acquired information. The studies have consistently found that secondary memory is the type of memory that is most negatively affected by increasing age.



    Answer A: Sensory memory is relatively unaffected by increasing age.

    • Answer C: Memory span (also known as primary memory) is an aspect of short-term memory and is relatively unaffected by increasing age.
    • Answer D: Implicit memory (memory that does not require conscious effort) is less affected than explicit memory by increasing age.
  144. Adolescent delinquency behavior is more likely among:

    A.males who report problematic substance abuse

    B.non-white youth regardless of trauma history

    C.boys with externalizing personality characteristics

    D.youth who have a history of trauma exposure and mental health conditions
    The correct answer is D.

    In a national representative sample, delinquency was more likely among youth with trauma history, mental health conditions, and substance use. [See Adams, Z. W., McCart, M. R., Zajac, K., Danielson, C. K., Sawyer, G. K., Saunders, B. E., & Kilpatrick, D. G. (2013). Psychiatric problems and trauma exposure in non-detained delinquent and nondelinquent adolescents. Journal of Clinical Child and Adolescent Psychology, 42(3), 323331.]

    Answer A: Problematic substance use was associated with delinquency in both genders in the above study.

    Answer B: Although there are racial disparities in delinquency, trauma exposure is a significant predictor of delinquency regardless of race, and it appears that trauma exposure may account for racial differences in delinquent behavior. [Adams, et al (2013), cited above, and Lopez, C. M., Andrews, A. R. III, Chisolm, A. M., de Arellano, M. A., Saunders, B., & Kilpatrick, D. G. (2017). Racial/ethnic differences in trauma exposure and mental health disorders in adolescents. Cultural Diversity and Ethnic Minority Psychology, 23(3), 382387.]

    Answer C: Personality factors have not been noted as a predictor of adolescent delinquency for either gender.
  145. In response to the cries of an infant:

    A.only the infant's mother exhibits an increase in heart rate and blood pressure

    B.only the infant's caregivers and older siblings exhibit an increase in heart rate and blood pressure

    C.only experienced parents exhibit an increase in heart rate and blood pressure

    D.most adults (both parents and non-parents) exhibit an increase in heart rate and blood pressure
    The correct answer is D.

    Most adults have an automatic physiological response to an infant's cries, with the pain cry producing the strongest response
  146. An infant with secure attachment will most likely respond to a stranger in the Strange Situation in which of the following ways?

    A.The infant will be friendly toward the stranger when mother is present but wary of the stranger when mother is absent.

    B.The infant will be wary of the stranger when mother is present but friendly toward the stranger when mother is absent.

    C.The infant will be friendly toward the stranger when mother is either present or absent.

    D.The infant will be wary of the stranger when mother is either present or absent.
    The correct answer is A.

    The Strange Situation was developed by Ainsworth and her colleagues (1978) who reported that securely attached infants were often friendly toward the stranger when mother was present but tended to be wary of the stranger when mother was absent, although some of these infants were somewhat comforted by the overtures of the stranger when separated from their mothers.

    Answer B: This answer does not describe the behavior associated with any of the three attachment patterns initially identified by Ainsworth.

    Answer C: In the Strange Situation, most infants with insecure-avoidant attachment reacted to the stranger in much the same way that they reacted to their mother and some were friendlier toward the stranger than their mother.

    Answer D: Infants with insecure-ambivalent attachment were wary of the stranger when their mother was either present or absent and were not calmed by the stranger when separated from their mothers.
  147. Generally speaking, the psychological adjustment of children with chronic illness is most dependent on:

    A.peer acceptance

    B.family attitudes toward the illness

    C.the child's personality type

    D.the child's level of functional disability
    The correct answer is D.

    If you are unfamiliar with the research in this area, the best approach to picking an answer would be to select the most general response. A number of studies have linked severity of the illness to outcome. However, some authors argue that it is not just the illness severity but the level of functional disability that determines the impact of the illness on the child [see, e.g., Garrison, W.T. and McQuiston, S. (1989). Chronic illness during childhood and adolescence. Newbury Park, CA: Sage].

    Answer A: Although peer acceptance is important for children and adolescents with chronic illness, it is not as important as the level of disability.

    Answer B: Family factors have been linked to outcomes in chronic illness but "family attitudes toward the illness" is too specific to be the correct response.

    Answer C:The research has not linked outcome to particular personality types
  148. Most children are not physically ready for toilet training until which age?

    A.9 to 12 months

    B.12 to 18 months

    C.20 to 24 months

    D.30 to 34 months
    The correct answer is C.

    The experts seem to vary somewhat with regard to this issue, but most consider 20 to 24 months a good time to begin toilet training.
  149. According to _____, gender role development involves developing conceptual frameworks (schemas) of masculinity and femininity as the result of sociocultural experiences.

    A.Bem

    B.Kohlberg

    C.Freud

    D.Mischel
    The correct answer is A.

    For the exam, be familiar with the gender role development theories of the four individuals listed in the answers to this question. Knowing that Bem's theory is known as "gender schema theory" would have helped you identify this as the correct answer.

    Answer B: Kohlberg theorized that the development of gender roles depends on a child grasping the concept that their sex remains fixed.

    Answer C: For Freud, gender role development takes place during the phallic stage, when the child's libido drives boys to experience the Oedipus complex and identify with their father and girls experience the Electra complex.

    Answer D: Mischel's theory suggested that observational learning is the basis for gender role development.
  150. Authoritative parenting, which has been associated with the most positive outcomes for children and adolescents, is best described as:

    A.high in control and high in responsiveness 

    B.high in control but low in responsiveness

    C.low in control and high in responsiveness 

    D.low in control and low in responsiveness
    The correct answer is A.

    Authoritative parents combine rational control with responsivity. Although they set clear rules and high standards for their children, they rely on reasoning, praise, explanations, and other inductive techniques to gain compliance, and they encourage independence. The offspring of authoritative parents tend to be assertive, socially responsible, and achievement-oriented, have high self-esteem, are self-confident, and usually obtain high grades in school.

    Answer B: Authoritative parents are responsive.

    Answer C: Authoritative parents do exercise control over their children.

    Answer D: This is the opposite of the authoritative parent.
  151. In females with ____________, part or all of one X chromosome is missing.

    A.Mullerian syndrome

    B.Fragile X syndrome

    C.Cushing syndrome

    D.Turner syndrome
    The correct answer is D.

    • Females with Turner syndrome have an XO chromosome pattern (i.e., one X chromosome with no other functioning sex chromosome). They do not develop secondary sex characteristics during puberty, are typically short with a thick, webbed neck and are usually infertile due to a lack of normal ovaries. Many have mild learning problems.
    •   
    • Answer A: Persistent Mullerian duct syndrome is a congenital disorder that is caused by a lack of anti-Mullerian hormone or receptors for this hormone.

    Answer B: Fragile X syndrome is a hereditary condition that causes an intellectual disability and other cognitive impairments as well as certain physical and behavioral abnormalities.

    Answer C: Cushing syndrome is due to malfunctioning of the adrenal cortex.
  152. The gross motor milestones for children _____ years of age include pedaling a tricycle, kicking a stationary ball forward, throwing a ball overhand, going up stairs using alternate feet, and running without falling.

    A.two

    B.three

    C.four

    D.five
    The correct answer is B.

    For the exam, be familiar with major developmental milestones. (Note that, because reported milestones for different ages vary somewhat in the literature, it's important to consider all of the milestones listed and not over-focus on any single milestone.) The motor skills listed in this question are gross motor milestones for three-year-old children.

    Answer A: Gross motor milestones for two-year-old children include climbing up on furniture, walking alone, walking up and down stairs one at a time, and catching a large ball using hands and chest.

    Answer C: Gross motor milestones for four-year-old children include hopping on one foot, standing on one leg, running to kick a ball, bouncing a ball, and going down stairs using alternate feet.

    Answer D: Gross motor milestones for five-year-old children include riding a scooter or bicycle, jumping rope, hopping on alternate feet, jumping over low obstacles, and catching a ball with both hands and arms bent.
  153. A young child exhibiting an insecure-disorganized attachment pattern reacts to the return of his/her mother in the Strange Situation with disorganized, conflicted, and, in some cases, self-injurious behaviors. This attachment pattern has been linked to parental abuse and neglect and with:

    A.elevated levels of cortisol in the child during the events of the Strange Situation

    B.lower-than-normal levels of cortisol in the child during the events of the Strange Situation

    C.elevated levels of cortisol in the child only when his/her mother returns during the Strange Situation

    D.elevated levels of cortisol in the mother (but not the child) during the events of the Strange Situation
    The correct answer is A.

    The levels of cortisol, a stress hormone released by the adrenal gland, differ for children with secure and insecure attachment to their primary caregivers. Securely attached children show an increase in cortisol levels when the caregiver leaves the Strange Situation, and the level of cortisol decreases when the caregiver returns. In contrast, in children exhibiting insecure-disorganized attachment, cortisol levels remain elevated throughout the course of the Strange Situation.[ Main, M. and Solomon, J. (1990). Procedures for identifying infants as disorganized/disoriented during the Ainsworth Strange Situation. In M. T. Greenberg, D. Cicchetti, and E. M. Cummings (Eds.). Attachment in the preschool years: Theory, research, and intervention (pp. 121-160). Chicago: University of Chicago Press].
  154. Research suggests that, in comparison to popular children, less popular children tend to be all of the following except:

    A.less intelligent

    B.less physically attractive

    C.less friendly and sociable

    D.less cooperative
    The correct answer is: C.

    Characteristics that are valued in the culture are the characteristics most associated with popularity during childhood. Unpopular children are not necessarily less friendly or sociable than popular children (Ascher and Hymel, 1981; Gottman, 1977).

    Answer A: Unpopular children are generally less intelligent than popular children.

    Answer B: Unpopular children tend to be less physically attractive than popular children.

    Answer D: Unpopular children do tend to be less cooperative than popular children.
  155. A 47-year old politician teaching his aides the essentials of dirty tricks tells them, "Nothing is wrong unless you get caught." One of his aides, a developmental psychologist, comments to another aide that the politician is apparently at the __________ level of moral development.

    A.preconventional

    B.conventional

    C.postconventional

    D.instrumental
    The correct answer is A.

    Kohlberg's levels of moral development are preconventional morality (punishment-obedience orientation and instrumental-relativist orientation), conventional morality (good boy/nice girl orientation and law and order orientation), and postconventional morality (legalistic orientation and universal ethical principles orientation). Individuals at the punishment-obedience level of preconventional morality believe an action is moral to the degree that it does not result in punishment. Although this stage is characteristic of children aged 4 to 10, it seems that this politician is also at this level.
  156. A securely attached infant:

    A.may or may not become upset when the parent leaves the room; is comforted by a stranger; and seeks contact with the parent on reunion

    B.becomes upset when the parent leaves the room; is unlikely to be comforted by a stranger; and seeks comfort from the parent on reunion

    C.becomes upset when the parent leaves the room; is comforted by a stranger; and may or may not seek contact with the parent on reunion

    D.becomes upset when the parent leaves the room; seeks comfort from a stranger if upset; and resumes play when the parent returns
    The correct answer is B.

    Research by Ainsworth and her colleagues identified four main patterns of attachment: secure, insecure/avoidant, insecure/ambivalent, and disorganized/disoriented. Securely attached children become upset when a parent leaves the room, are not comforted by a stranger, and, upon reunion with the parent, seek contact and comfort but gradually return to play.

    Answer A: An insecure/ambivalent infant is likely to react to their mothers and strangers in a similar manner - feeling comforted by a stranger.

    Answer C: An insecure/ambivalent infant is likely to react to their mothers and strangers in a similar manner - feeling comforted by a stranger, as well as feeling ambivalent when she returns. 

    Answer D: An insecure/ambivalent infant is likely to react to their mothers and strangers in a similar manner - feeling comforted by a stranger.
  157. Jerome Kagan has related the basic temperament quality of inhibition (degree of approach or withdrawal in new situations) to which of the following?

    A.prenatal development

    B.intellectual capacity

    C.caregiver behaviors

    D.CNS reactivity
    The correct answer is D.

    Infants with a high degree of behavioral inhibition also exhibit high CNS activity (especially in the amygdala and hypothalamus) in new situations. [See Kagan, J. et al.(1990). The temperamental quality of inhibition and lack of inhibition. In M. Lewis and S. M. Miller (Eds.). Handbook of developmental psychopathology. New York: Plenum Press. (pp. 219-226).]
  158. Patterson and his colleagues focus on the family in describing the origins of aggressiveness in children. Specifically, they relate high aggressiveness in children to:

    A.attachment insecurity in infancy

    B.parental permissiveness and overindulgence

    C.parental rejection and lack of warmth

    D.coercive exchanges between parent-child
    The correct answer is D.

    Gerald Patterson and his colleagues take a social learning approach to explain aggressive behavior in children and adolescents. His "coercion hypothesis" focuses on interactions between parents and their children, and identifies the parents' childrearing skills as a key determinant of the nature of these interactions. [e.g. Smith, J. D., Dishion, T. J., Shaw, D. S., Wilson, M. N., Winter, C. C., & Patterson, G. R. (2014). Coercive family process and early-onset conduct problems from age 2 to school entry. Development and psychopathology, 26(4 Pt 1), 917932.]

    Answer A: Patterson does not focus on attachment insecurity.

    Answer B: Permissiveness and indulgence is not a focus of Patterson's research.

    Answer C: Parental rejection and lack of warmth is not a focus of Patterson's research.
  159. Researchers interested in the effects of aging on autobiographical memory use the term "reminiscence bump" to describe the tendency of adults aged 65 and older to have the largest number of memories for events that occurred when they were between the ages of:

    A.10 and 30

    B.5 and 15

    C.25 and 45

    D.40 and 55
    The correct answer is A.

    Older adults typically report the largest number of memories for events that occurred in the ten-year period prior to being evaluated. The next largest number of memories is for events that occurred during adolescence and early adulthood, which is called, "the reminiscence bump".

    Answer B: The childhood period is not the most recalled era for older adults.

    Answer C: The reminiscence bump includes recall for the events of adolescence as well as young adulthood, and therefore this is not the correct answer.

    Answer D: Although memory for recent events may make the period of middle age more likely to be recalled, the term reminiscence bump specifically refers to the tendency to recall events occurring in adolescence and early adulthood.
  160. Following a stroke, a 71-year-old woman exhibits loss of vision in her right visual field. Most likely, this impairment is the result of damage to the

    A.visual cortex in her left hemisphere

    B.visual cortex in her right hemisphere

    C.left posterior temporal lobe

    D.right posterior temporal lobe
    The correct answer is A.

    Vision is mediated by the visual cortex. Most of the body's functions-including vision-are mediated by the opposite side of the brain. Therefore, loss of vision in the right visual field would be caused by damage in the left hemisphere.
  161. What percent of neurons are lost by the age of 70?

    A.15

    B.1

    C.40

    D.5
    The correct answer is B.

    Although neurons die at a rate of about 10,000 per day, less than 2% of neurons will be lost by the age of 70. Also, neurons regenerate through the lifespan in certain parts of the brain (neurogenesis).
  162. Hemiplegia affecting the right side of the body suggests damage to the

    A.motor cortex in the right hemisphere

    B.motor cortex in the left hemisphere

    C.somatosensory cortex in the left hemisphere

    D.somatosensory cortex in the right hemisphere
    The correct answer is B.

    Hemiplegia refers to partial or complete paralysis on one side of the body. The right side of the body is controlled by the left side of the brain. Damage to the motor cortex could result in paralysis. Since the paralysis is on the right side of the body, this implies that the damage is to the motor cortex in the left hemisphere.
  163. Combining an MAOI with certain cold, cough, sinus, or allergy medication can produce which of the following?

    A.neuroleptic malignant syndrome

    B.hypertensive crisis

    C.agranulocytosis

    D.tardive dyskinesia
    The correct answer is B.

    An MAOI in combination with certain drugs can produce a hypertensive crisis (noradrenergic syndrome), which involves a severe headache, diaphoresis, elevated blood pressure, neck stiffness, and neuromuscular agitation; or a hyperpyrexic crisis (serotonin syndrome), which is characterized by hyperthermia, tachycardia, nausea, shivering, restlessness, confusion, and insomnia.

    Answer A: Neuroleptic malignant syndrome is a side effect of traditional antipsychotics.

    Answer C: Agranulocytosis is a decrease in a certain white blood cell and can be a side effect of atypical antipsychotics.

    Answer D:  Traditional antipsychotics can produce tardive dyskinesia.
  164. _______ reduce or block the effects of a neurotransmitter.

    A.Antagonists

    B.Agonists

    C.Partial agonist

    D.Inverse agonist
    The correct answer is A.

    Antagonist drugs produce no activity in cells on their own. Instead, they block or reduce the effects of a neurotransmitter (agonist). Direct antagonists attach to a neurotransmitter's receptor site and indirect antagonists attach to a site other than the one being used by the neurotransmitter in question, thus interfering with the action of the neurotransmitter.

    Answer B: Agonists facilitate the action of a neurotransmitter, either by mimicking the effect of that neurotransmitter at a receptor site or by binding on a receptor cell.

    Answer C: Partial agonists produce effects similar to, but less effective than, a neurotransmitter.

    Answer D: Rather than reducing or blocking, an inverse agonist produces the opposite effect of a neurotransmitter.
  165. The "therapeutic window" refers to __________.

    A.the time when a drug is effective to treat an injury

    B.the time when clinical interventions can be used prior to turning to drug treatment

    C.the time frame when the most effective progress can occur from treatment for neurological disorders, usually within 2 weeks from onset of symptoms

    D.the time frame when therapy combined with drug treatment is most effective
    The correct answer is A.

    The therapeutic window refers to the time between injury and drug treatment during which the treatment is still effective. Monitoring of individuals under drug treatment is recommended to ensure the therapeutic window and to treat any unusual reactions to a medication.
  166. Which area of the brain do barbiturates depress that is involved in behavioral alerting, attention, and arousal responses?

    A.brain stem

    B.amygdala

    C.hypothalamus

    D.hippocampus
    The correct answer is A.

    The brain stem is important in the regulation of respiration, blood pressure, heart rate, GI functioning, and the states of sleep and wakefulness. Depressant drugs (i.e., barbiturates) interrupt impulses to the reticular activating system, which is located in the brain-stem, resulting in decreased arousal responses and attentiveness.

    Answer B: The amygdala is a component of the limbic system and plays an important role in emotions and behavior.

    Answer C: The hypothalamus is involved in daily activities like eating and drinking, in the control of the body's temperature and energy maintenance, and stress control. It also modulates the endocrine system through connections with the pituitary gland.

    Answer D: The hippocampus is embedded within the temporal lobe. It regulates motivation, emotion, learning, and memory.
  167. Prolonged exposure to stress may lead to chronic ________ by the adrenal glands.

    A.suppression of cortisol release

    B.secretion of oxytocin

    C.suppression of oxytocin release

    D.secretion of cortisol
    The correct answer is D.

    Knowing that cortisol is secreted by the adrenal cortex would have helped you answer this question. Cortisol and the other corticosteroids are collectively known as the "stress hormones." Chronic stress leads to chronic secretion of these hormones, which can cause a number of problems including increased susceptibility to infection.

    Answer A: This is the opposite of true; under chronic stress the adrenal glands secrete cortisol.

    Answers B and C: Oxytocin is secreted by the pituitary gland and, in females, controls uterine contractions and milk production.
  168. Tactile sensation and visuospatial functions are mediated primarily by the

    A.frontal lobe

    B.parietal lobe

    C.occipital lobe

    D.temporal lobe
    The correct answer is B.

    The parietal lobe receives tactile input and mediates visuospatial functions, reading, and calculation.

    Answer A: The frontal lobe mediates motor functions, language production, personality, and executive cognitive functions.

    Answer C: The occipital lobe is responsible for vision and visual perception.

    Answer D: The temporal lobe is responsible for audition, language, comprehension, and memory.
  169. The excess of which neurotransmitters can be a potent neuronal "excitotoxin," triggering seizures, brain damage, or other neurodegenerative disorders?

    A.glutamate

    B.dopamine

    C.norepinephrine

    D.serotonin
    The correct answer is A.

    Glutamate excess can be a potent neuronal "excitotoxin," triggering either rapid or delayed death of neurons. This can lead to seizures and stroke-related brain damage, Huntington's disease, and Alzheimer's.

    Answer B: Dopamine is a neurotransmitter that is used by the nervous system to send messages between cells. It plays a role in how individuals feel pleasure.

    Answer C: Norepinephrine is both a hormone produced by the adrenal gland and a neurotransmitter, which transmits signals across nerve endings. The general function of norepinephrine is to mobilize the brain and body for action.

    Answer D: Serotonin is both a hormone and neurotransmitter that stabilizes mood. It is also involved in eating, sleeping, memory, and learning.
  170. Which of the following is the most appropriate drug treatment for alcoholism?

    A.Disulfiram

    B.Concerta

    C.Demerol

    D.Phenobarbital
    The correct answer is A.

    Disulfiram (Antabuse) is a drug that inhibits alcohol metabolism. This causes an accumulation of unpleasant reactions when one drinks (e.g. nausea, vomiting, sweating, headache, etc.). that help deter the person from drinking alcohol in the future.

    Answer B: Concerta is a common psychostimulant used to treat ADHD in children and adults.

    Answer C: Demerol is a pure synthetic type of narcotic-analgesic. These drugs are used as treatments for pain management.

    Answer D: Phenobarbital is a sedative-hypnotic barbiturate that is used as a sedative or to induce sleep. Since alcohol and barbiturates both have a depressive and sedative effect on the body, this would not be an effective drug for alcohol treatment.
  171. A person with damage to the hippocampus and adjacent areas in the temporal lobes will most likely demonstrate which of the following?

    A.loss of motor memory

    B.impairments in working memory

    C.inability to acquire a classically-conditioned response

    D.inability to form long-term memories about facts and events
    The correct answer is D.

    The hippocampus and adjacent areas in the temporal lobe are involved in memory. The effects of lesions in these areas of the brain include profound impairments in the ability to form new memories of facts and events.

    Answer A: Motor memory can be created by the cerebellum.

    Answer B: Damage to the prefrontal cortex can result in impairments with working memory.

    Answer C: Even with damage to the hippocampus, the cerebellum allows one to create implicit memories, such as those made through classical conditioning.
  172. Research on the etiology of Tourette's syndrome has linked it to excessive activity of dopamine receptors in the

    A.amygdala

    B.septum

    C.caudate nucleus

    D.medulla oblongata
    The correct answer is C.

    Tourette's syndrome involves involuntary tics and, not surprisingly, the area of the brain associated with automatic movements has been implicated in this disorder. The caudate nucleus mediates automatic movement, and super sensitivity of dopamine receptors in this structure has been linked to Tourette's syndrome.
  173. Tricyclics are used to treat all of the following disorders except _________.

    A.premenstrual dysphoric disorder

    B.depression

    C.panic disorder

    D.bulimia
    The correct answer is A.

    SSRIs are commonly used to treat women with PMDD.
  174. Up to 50% or more of individuals who experience a minor brain injury develop post-concussion syndrome. The majority of these individuals fully recover within ______. However, for those whose symptoms last for more than _________, symptoms may be permanent.

    A.one to three weeks; six months

    B.one to two months; nine months

    C.three to six months; two years

    D.one to three months; one year
    The correct answer is D.

    Post-concussion syndrome is a pattern of somatic and psychological symptoms that occur in 50% or more of individuals who experience a minor brain injury. Most individuals who develop minor symptoms following a brain injury fully recover within one to three months after the injury. A minority of individuals continue to experience symptoms for more than one year and, for these individuals, symptoms may be lifelong.
  175. Specific to benzodiazepines, the drug abuse potential, behavioral-rewarding effects, and psychological dependency result from actions on ________ receptors.

    A.serotonin

    B.dopamine

    C.GABA

    D.acetylcholine
    The correct answer is C.

    All benzodiazepines facilitate the inhibitory action of GABA binding. The behavioral-rewarding effects, drug abuse potential, and psychological dependency probably result from actions on GABA receptors that modulate the discharge of neurons located in the ventral tegmentum and the nucleus accumbens.

    Answer A: Serotonin is a neurotransmitter that relays signals between nerve cells, or neurons, regulating their intensity. It plays a key role in the central nervous system, the general functioning of the body, and the gastrointestinal tract.

    Answer B: Dopamine is a neurotransmitter that plays a role in pleasure, motivation, and learning.

    Answer D: Acetylcholine is the primary neurotransmitter of the parasympathetic nervous system, the part of the autonomic nervous system that contracts smooth muscles, dilates blood vessels, increases bodily secretions, and slows the heart rate.
  176. Which of the following conditions is not described accurately?

    A.Multiple sclerosis is a musculoskeletal syndrome that produces muscular pain, stiffness, and spasms and is often accompanied by fatigue, gastrointestinal disorders, and depression.

    B.ALS is a progressive peripheral nerve disorder that causes muscle weakness, tremor, and spasticity.

    C.Myasthenia gravis is a neuromuscular autoimmune disorder that produces weakness in the eye and facial muscles, difficulty swallowing, and slurred speech.

    D.Huntington's disease is a hereditary disease that causes involuntary jerky movements, impaired memory and judgment, and depression.
    The correct answer is A.

    The conditions described in the answers to this question are neurological disorders that cause impairment in sensory and/or motor functioning. Note that the question is asking which condition is NOT accurately described. The symptoms presented in this answer describe fibromyalgia rather than multiple sclerosis (MS). MS is caused by demyelination of nerve fibers in the brain and spinal cord and is believed to be due to an abnormal immune system response. Its primary symptoms include impairments in sensory and motor functioning (e.g., paresthesias, shooting pain in the back and limbs, problems with balance, and muscle weakness); deficits in memory, reasoning, and concentration; sexual dysfunction; and vision abnormalities.

    Answer B: This correctly describes ALS (amyotrophic lateral sclerosis), which is also known as Lou Gehrig's disease and is a degenerative disease that eventually leads to a loss of muscle control and paralysis.

    Answer C: The term myasthenia gravis means "grave muscle weakness". This disorder can affect any of the body's muscles, but the muscles that control the eyes, facial expression, chewing, and swallowing are ordinarily most affected.

    Answer D: This answer accurately describes Huntington's disease, which produces a combination of motor, cognitive, and psychiatric symptoms.
  177. The anterior cingulate cortex is believed to be involved in

    A.color vision.

    B.spatial memory.

    C.emotional reactions to pain.

    D.the physiological component of anxiety.
    The correct answer is C.

    The anterior cingulate cortex is located in the limbic cortex on the medial side of the cerebral hemispheres. A number of studies have confirmed that exposure to painful stimuli activates the anterior cingulate cortex. However, this structure does not appear to be involved in the perception of pain, but rather, in the emotional reaction to painful stimuli.

    Answer A: The visual cortex in the occipital lobe handles color vision.

    Answer B: The hippocampus is a key area for spatial memory.

    Answer D: The limbic system, brain stem, and frontal lobe are all involved in anxiety. The brainstem is primarily involved in the physiological components of anxiety (e.g., heart rate, breath, body temperature, etc.).
  178. Which of the following is least likely to be an initial symptom of multiple sclerosis?

    A.impaired memory and attention

    B.fatigue that worsens in the afternoon

    C.blurred or double vision

    D.heaviness, weakness, or loss of sensation in the legs
    The correct answer is A.

    Multiple sclerosis (MS) is caused by demyelination of nerve fibers in the central nervous system. Although its symptoms vary from individual to individual, the progression of symptoms usually follows a predictable pattern. It is estimated that up to 70% of people with MS develop some type of cognitive dysfunction, and this most often involves impairments in attention, recent memory, and information processing speed. While some of these individuals experience cognitive problems in the early stages of the disorder, the majority do not do so until they have had the disease for an extended period of time. Therefore, of the answer options provided, this is the best choice.

    Answer B: Fatigue (especially fatigue that worsens in the afternoon) is a common initial symptom of this disorder.

    Answer C: Visual problems are a common initial symptom of MS.

    Answer D: Sensory and motor problems are initial signs of MS and most often affect the legs.
  179. Which of the following plays a role in regulating body temperature?

    A.hippocampus and hypothalamus

    B.hypothalamus and thyroid

    C.basal ganglia and thalamus

    D.hindbrain and reticular activating system
    The correct answer is B.

    The hypothalamus regulates hunger, thirst, sleep, sex, emotional reactions, and body temperature. The thyroid gland secretes thyroxine. Too much thyroxine leads to an elevated body temperature (a symptom of hyperthyroidism) and too little thyroxine leads to lowered body temperature (a symptom of hypothyroidism).

    Answer A: The hypothalamus does regulate body temperature but the hippocampus does not (it is implicated in learning, memory, and emotional regulation), thus, this answer is not correct.

    Answer C: The basal ganglia are involved in planning, organizing, voluntary movement, and emotions. The thalamus affects motor activity, memory, language, and relays information to the appropriate areas of the brain.

    Answer D: The hindbrain coordinates functions like sleep, wakefulness, respiration, and motor activity. The reticular activating system is vital to consciousness, arousal, and wakefulness.
  180. Synesthesia is best conceptualized as

    A.a learned phenomenon.

    B.the result of imagination.

    C.a physical process in the brain.

    D.a manifestation of the early stages of psychosis.
    The correct answer is C.

    Synesthesia occurs when two or more senses are automatically and involuntarily joined so that stimulation of one sense automatically elicits another (e.g., the sound of a voice elicits a specific color). Synesthesia is believed to be caused by brain functioning. One theory attributes it to unusually dense connections between sensory organs of the brain.
  181. Your new client, Annabelle, reports that she has felt fidgety and anxious and has been unable to sleep for the past few weeks. She also complains of heart palpitations, weight loss, and inability to focus on her schoolwork. Despite the cold weather, she is wearing a lightweight dress without a coat or sweater. Annabelle's symptoms are most suggestive of which of the following?

    A.hypomania

    B.hyperthyroidism

    C.Raynaud's disease

    D.hypoglycemia
    The correct answer is B.

     Hyperthyroidism is characterized by a sped up metabolism, elevated body temperature, weight loss, general excitability, nervousness, insomnia, and accelerated heart rate. The client's symptoms are characteristic of hyperthyroidism which is caused by an overactive thyroid gland.

    Answer A: A person with hypomania does display excitability and nervousness but does not exhibit such physical signs as increased body temperature and heart rate.

    Answer C: This alternative is a distractor. Raynaud's disease involves restricted blood flow to the fingers or toes and is characterized by pallor, paresthesias, and pain.

    Answer D: Hypoglycemia is characterized by hunger, dizziness, and depression.
  182. According to the gate-control theory of pain, when __________ information is received, cells in the spinal cord act as a gate that blocks some incoming pain signals.

    A.important 

    B.too much 

    C.very little

    D.insignificant
    The correct answer is B.

    According to the gate-control theory of pain (Melzack, 1973), the nervous system can process only a limited amount of sensory information at any one time. When too much information is being received, cells in the spinal cord act as a gate that blocks some incoming pain signals.
  183. The onset of puberty occurs when certain cells in the __________ secrete gonadotropin-releasing hormones.

    A.reticular formation

    B.thalamus

    C.hippocampus

    D.hypothalamus
    The correct answer is D.

    At puberty, the gonads (testes and ovaries) produce hormones that are responsible for sexual maturation. The secretion of the gonadotropin-releasing hormones by the hypothalamus stimulates the production and release of the gonadotropic hormones by the pituitary gland, which then stimulates the gonads to release sex hormones.

    Answer A: The reticular formation plays a role in promoting arousal and consciousness. This function is mediated by the reticular activating system (RAS), also known as the ascending arousal system.

    Answer B: The thalamus is composed of different nuclei that serve roles ranging from relaying sensory and motor signals, as well as the regulation of consciousness and alertness.

    Answer C: The hippocampus is part of the limbic system that is involved in the formation of new memories and is associated with learning and emotions.
  184. Which of the following is a type of nonfluent aphasia that involves deficits in both receptive and expressive language?

    A.conduction aphasia

    B.global aphasia

    C.anomic aphasia

    D.transcortical motor aphasia
    The correct answer is B.

    Global aphasia is a severe form of nonfluent aphasia, caused by damage to the left side of the brain, that affects both receptive and expressive language skills as well as auditory and visual comprehension.

    Answer A: Conduction aphasia is a type of fluent aphasia that involves phonemic paraphasias (substitutions of one phoneme for another) and impaired speech repetition.

    Answer C: Anomic aphasia is a type of fluent aphasia that is characterized by anomia with normal comprehension and speech repetition.

    Answer D: Transcortical motor aphasia is a type of nonfluent aphasia with normal auditory comprehension and speech repetition.
  185. _______ are alternative assumptions and patterns of thinking relevant at a specific historical time.

    A.Competencies

    B.Traits

    C.Personality

    D.Paradigms
    The correct answer is D.

    A paradigm consists of a set of assumptions, ways of thinking, attitudes, concepts, values, procedures, and techniques that constitute a generally accepted theoretical framework.

    Answer A: Competencies are a set of behavior patterns in which someone needs to bring a job or other situation to be able to undertake tasks or functions.

    Answer B: Traits are any persistent characteristic (e.g., emotional, cognitive, behavioral), which influences the way in which personality is demonstrated.

    Answer C: Personality refers to individual differences in characteristic patterns of thinking, feeling, and behaving.
  186. Which of the following assists a clinician in administering the subtests of the Woodcock-Johnson IV in knowing when to stop a subtest due to an examinee's ability to respond correctly?

    A.Halo effect

    B.Basal rules

    C.Primacy effect

    D.Ceiling rules
    The correct answer is D.

    The ceiling level is the set of consecutive items above which the set examinee has a 0% chance of responding correctly. Ceiling rules enable the clinician to know when to discontinue a subtest based on an examinee's ability.

    Answer A: Halo effect is a cognitive bias that occurs when an initial positive judgment about a person unconsciously colors the perception of the individual.

    Answer B: Basal rules enable a clinician to establish a start point so that every subtest item is not unnecessarily administered.

    Answer C: Primacy effect is the tendency for individuals without neurological impairment to show enhanced memory for items presented at the beginning of a list relative to items presented in the middle of the list.
  187. Which of the following instruments would be appropriate to assess the severity of stereotyped behaviors and communication impairment in a 4-year old?

    A.Gilliam Autism Rating Scale-Second Edition (GARS-2)

    B.Conners' Kiddie Continuous Performance Test, Version 5 (K-CPT)

    C.Millon Pre-Adolescent Clinical Inventory (M-PACI)

    D.NEPSY-II
    The correct answer is A.

    The GARS-2 is an observer-report standardized rating scale for Autistic Disorder including a norm-referenced sample ranging in age from 3 to 22 years. It includes an Autistic Disorder Index and subscales scores, which measure the severity of stereotyped behaviors, communication impairment, and social impairment.

    Answer B: The K-CPT is normed on individuals aged 4 to 5 years and used in determining the likelihood that an individual manifests a disorder of attention.

    Answer C: The M-PACI is a self-report personality inventory normed on preadolescents aged 9 to 12 years. It consists of validity scales and diagnostic scales most relevant to clinical disorders, personality disorders, and impaired intellectual functioning.

    Answer D: The NEPSY-II is a neuropsychological test battery normed on children and adolescents from age 3 to 16 years that measures areas including attention, executing functioning, language, memory and learning, sensorimotor, social perception, and visuospatial processing.
  188. Which of the following is a multivariate data analysis method for finding the linear combination of variables that best describes the classification of groups into discrete categories?

    A.Factor analysis

    B.Discriminant analysis

    C.Descriptive statistics

    D.Item analysis
    The correct answer is B.

    Discriminant analysis finds a set of prediction equations based on independent variables that are used to classify individuals. Discriminant analysis parallels multiple regression analysis.

    Answer A: Factor analysis is a method of finding the minimum number of dimensions (characteristics, attributes), called factors, to account for a large number of variables.

    Answer C: Descriptive statistics are methods used to provide a concise description of a collection of quantitative information.

    Answer D: Item analysis is a set of methods used to evaluate test items. The most common techniques involve the assessment of item difficulty and item discriminability.
  189. Which of the following are tests designed to assess the examinee's potential for learning or their ability to perform in an area without specific training?

    A.Cognitive tests

    B.Behavioral observation tests

    C.Aptitude tests

    D.Personality tests
    The correct answer is C.

    Aptitude tests are used to determine an individual's propensity to succeed at a given activity.

    Answer A: Cognitive tests measure the examinee's mental capacities, such as general mental ability tests, intelligence tests, and academic skills.

    Answer B: Behavioral observation tests are tests that involve observing an individual's behavior to learn how they typically respond in a particular context.

    Answer D: Personality tests are designed to measure human character or disposition.
  190. Which type of distribution has its mean at the highest point and is symmetrical about the mean?

    A.Binomial distribution

    B.Negatively skewed distribution

    C.Positively skewed distribution

    D.Normal distribution
    The correct answer is D.

    For a normal distribution the mean, median, and mode coincide at the center. The curve is perfectly symmetrical and asymptotic at its extremes.

    Answer A: A binomial distribution is a probability distribution that summarizes the likelihood that a value will take one of two independent values under a given set of parameters of assumptions.

    Answer B: A negatively skewed distribution refers to the distribution type where more values are plotted on the right side of the graph and the tail of the distribution is longer on the left.

    Answer C: A positively skewed distribution is a type of distribution in which most values are clustered around the left tail of the distribution while the right tail is longer.
  191. Which of the following is the negative effect an unfair and biased selection procedure has on a protected class?

    A.Adverse impact

    B.Criterion-referencing

    C.Cut-off score

    D.Confidence interval
    The correct answer is A.

    An adverse impact results from practices that seem to be neutral but that disproportionately and negatively affect protected groups.

    Answer B: The purpose of a criterion-referenced test is to measure an individual's absolute level of accomplishment of the material covered in that exam according to some standard reference point.

    Answer C: Cut-off scores are values that mark the lowest point at which a certain status or category is attained.

    Answer D: A confidence interval is a range of values that is likely to include a population value with a certain degree of confidence.
  192. Which of the following is a measure of interrater reliability that adjusts for the level of chance agreement between raters' scores?

    A.Predictive validity

    B.Magnitude of effect

    C.Kappa coefficient

    D.Standard error of measurement
    The correct answer is C.

    The Kappa coefficient is a statistical measure of inter-rater reliability used to assess qualitative documents and determine the reliability between two raters.

    Answer A: Predictive validity is the degree to which a measure can predict another measure given at a later time.

    Answer B: The magnitude of the effect is a measure of the strength of the relation between variables, or the magnitude, of change across time, or the magnitude of difference between groups.

    Answer D: The standard error of measurement estimates how repeated measures of an individual on the same instrument tend to be distributed around their "true" score.
  193. _______ is the degree to which an obtained measure of effect has a practical value or can guide clinical judgments.

    A.Diagnostic efficiency

    B.Clinical utility

    C.Clinical significance

    D.Likelihood ratio
    The correct answer is C.

    • Clinical significance is the degree to which measures contribute meaningful information and aid clinical judgment.
    •   
    • Answer A: Diagnostic efficiency is the degree to which established test cut-off scores accurately identify persons diagnosed with or without a disorder, as identified by an external criterion.

    Answer B: Clinical utility is the degree to which the results of an instrument assist the clinician in making judgments about a client or enhance the validity of those judgments.

    Answer D: The likelihood ratio is the degree to which the odds of a correct classification are increased by the use of the assessment data.
  194. The unavailability of preventive health services in minority communities, a lack of quality health care in close proximity to these communities, and the delivery of substandard health care services to members of these communities is best attributed to which of the following:

    A.subtle racism

    B.institutional racism

    C.personally mediated racism

    D.internalized racism
    The correct answer is B.

    "Institutional racism" refers to denial or restriction of material conditions (e.g., access to health care) and access to power to members of minority groups. Several types of racism have been identified. C. P. Jones (2000) defines three levels of racism - institutional, personally mediated, and internalized [Levels of racism: A theoretic framework and a gardener's tale, American Journal of Public Health, 90(8), 1212-1216, 2000].

    Answer A: "Subtle racism" refers to a less blatant (more covert) form of racism, which some experts contend has replaced "old-fashioned" (overt) prejudice and discrimination. The term subtle racism is used most often to describe the beliefs, attitudes, and actions of individuals (rather than institutions).

    Answer C: "Personally mediated racism" refers to prejudice and discrimination at the individual level.

    Answer D: "Internalized racism" refers to "acceptance by members of the stigmatized races of negative messages about their own abilities and intrinsic worth" (p. 1213).
  195. In team sports, the "home advantage" phenomenon has most consistently been linked to:

    A.the demoralization of the opposing team

    B.the impact of a supportive home audience

    C.the players expectations

    D.the players familiarity with the home court
    The correct answer is B.

    If you're a sports fan, you probably know that the home advantage refers to the tendency of sports teams to win more often when they are playing on their home court. The most consistently supported explanation is that it is due to the social support that fans give the home team. (K. S. Courneya and A. V. Carron, The home advantage in sports competitions: A literature review, Journal of Sport & Exercise Psychology, 14, 13-27, 1992.)
  196. Advocates for prayer in school hope to persuade other members of the community to support their position. According to "social judgment theory," other community members are most likely to be convinced if the discrepancy between their initial position on this issue and the position of the advocates of school prayer is:

    A.is low

    B.is moderate

    C.is high

    D.varies from person to person
    The correct answer is B.

    According to "social judgment theory, a person's potential for attitude change can be described in terms of three dimensions or latitudes: latitude of acceptance, latitude of non-commitment, and latitude of rejection. When an attitude is within the person's latitude of acceptance, that person already agrees with the position, so no attitude change occurs. When it is within the person's latitude of rejection, the person is unlikely to change his/her attitude. Therefore, the greatest attitude change occurs when the attitude is within the person's latitude of non-commitment, which occurs when there is a moderate discrepancy between the person's initial position and the position of the communication.
  197. A listener is most likely to process a persuasive message peripherally (i.e., use the "peripheral route") if she is:

    A.bored

    B.in a good mood

    C.in a bad mood

    D.emotionally unstable
    The correct answer is B.

    This question is asking about the "elaboration likelihood model." According to this model, whether a person uses the central route (carefully considers an argument) or peripheral route (relies on heuristic criteria) depends on several factors, including the nature of the message and the person's mood. A person is more likely to rely on the peripheral route when he/she is in a good mood.
  198. Sherif (1935) used the 'autokinetic effect' to study which of the following phenomenon:

    A.conformity

    B.affiliation

    C.psychological reactance

    D.locus of control
    The correct answer is A.

    The apparent movement of a stationary light in a dark room is referred to as the "autokinetic effect." Sherif (1935) used the autokinetic effect to study conformity. He found that estimates of movement varied widely when a participant made an estimate alone but conformed to the estimates of other people when he/she was asked to give an estimate after hearing the estimates of others.

    Answer B: While many researchers have studied affiliation, Sherif is not associated with the concept. 

    Answer C: Brehm is most closely associated with psychological reactance.

    Answer D: Bandura is most closely associated with locus of control and self-efficacy.
  199. The "sleeper effect" (Hovland & Weiss, 1952) is useful for understanding:

    A.why people might reject a persuasive message immediately after it is delivered by a low credible communicator but accept the message six weeks later

    B.why people are more likely to accept a persuasive message that they accidentally overheard than a message that was intentionally delivered to them

    C.why a decision-maker chooses the first solution that meets minimum criteria of acceptability rather than obtaining the information needed to choose the optimal solution

    D.why group decisions are often more extreme (riskier or more conservative) than the decision each group member would make alone
    The correct answer is A.

    The "sleeper effect" refers to the delayed impact of a persuasive message when the message is accompanied by a discounting cue (e.g., is delivered by a low credible communicator). Hovland & Weiss (1952) found that, over time, the persuasive impact of a message delivered by a highly credible communicator diminished while the persuasive impact of the same message delivered by a low credible communicator increased. They referred to the latter phenomenon as the "sleeper effect."

    Answer B: Although accidental (versus intentional) messages are often more readily accepted, this phenomenon is not referred to as the sleeper effect.

    Answer C: This answer refers to the bounded rationality model of decision making which states that decision-makers often "satisfy" rather than "optimize" due to limited time and resources.

    Answer D: This answer describes group polarization.
  200. __________ theory predicts that we tend to like others whose initially negative feelings toward us change to positive feelings more than we like those who have positive feelings toward us from the beginning.

    A.Self-verification 

    B.Gain-loss 

    C.Social exchange 

    D.Balance
    The correct answer is B.

    "Gain-loss theory" predicts that we tend to like others whose initial negative feelings toward us change to positive feelings more than those whose feelings are consistently positive and, conversely, that we tend to dislike others whose initially positive feelings toward us change to negative feelings more than those who have negative feelings toward us from the beginning.

    Answer A: "Self-verification theory" predicts that people prefer others who provide them with accurate information about themselves (i.e., with information that is consistent with their self-evaluations).

    Answer C: "Social exchange theory" focuses on the rewards and costs of a relationship and predicts that we are more likely to remain in a relationship when the rewards exceed the costs.

    Answer D: "Balance theory' uses the concept of cognitive consistency to explain attitude change and focuses on the relationships between three entities - the person, another person, and a third person or an idea, event, or object.
  201. Kahneman and Tversky's (1979) notion of __________ predicts a tendency for a loss of a certain magnitude to seem more aversive to a person than a gain of the same magnitude seems attractive.

    A.status quo bias

    B.psychological reactance

    C.loss aversion

    D.gain-loss theory
    The correct answer is C.

    Kahneman and Tversky's notion of "loss aversion" is part of their cumulative prospect theory and refers to the tendency to weigh losses more heavily than gains. It proposes that people experience losses more intensely than gains of the same magnitude and, consequently, are unlikely to take risks.

    Answer A: The "status quo bias" refers to the tendency to maintain the status quo, because the disadvantages of changing are perceived to be greater than the advantages.

    Answer B: "Psychological reactance" occurs when an attempt at social control causes a person to feel a loss of freedom and, in response, reacts in a way that is the opposite of what is requested or desired.

    Answer D: "Gain-loss theory" predicts that a complement will be valued more when it comes from a critic than from someone who has bestowed compliments in the past. (D. Kahneman & A. Tversky, Prospect theory: An analysis of decisions under risk, Econometrika, 47, 263-291, 1979.)
  202. Byrne's "law of attraction" emphasizes the role of _________ as an essential determinant of our attraction to others.

    A.physical attractiveness

    B.reciprocal liking

    C.propinquity

    D.attitude similarity
    The correct answer is D.

    According to Byrne, people who have similar attitudes to us reinforce our perceptions of the world and, consequently, are associated with positive feelings that increase attraction.
  203. The tendency to overestimate personality traits as the cause of behavior and underestimate the influence of situational factors is referred to as:

    A.the halo bias

    B.the discounting principle

    C.the self-serving bias

    D.the fundamental attribution bias
    The correct answer is D.

    The tendency for an observer to attribute the behavior of another person to dispositional factors, as opposed to situational ones, is known as the fundamental attribution bias.

    Answer A: The halo bias is a type of cognitive bias in which your overall impression of a person influences how you feel and think about their character.

    Answer B: The discounting principle refers to how someone attributes a cause to an eventual outcome. 

    Answer C: The tendency to blame external factors for your failures and take credit for your successes is referred to as the self-serving bias.
  204. A developmental psychologist finds that low birthweight is associated with a higher risk for ADHD in boys but not girls. In this situation, gender is a(n)_____.

    A.mediator variable

    B.moderator variable

    C.feedback variable

    D.outcome variable
    The correct answer is B.

    In this situation, gender is a moderator variable because it affects (moderates) the strength of the relationship between the independent and dependent variables (i.e., between birthweight and risk for ADHD).

    Answer A: A mediator variable is responsible for an observed relationship between an independent variable and a dependent (outcome) variable. In other words, the independent variable affects the mediator variable which, in turn, affects the dependent variable.

    Answer C: Feedback variable is a distractor term used in engineering and is not relevant to the situation described in this question.
  205. Researchers use structural equation modeling (SEM) to_____.

    A.assess the length of time to the occurrence of a critical event

    B.evaluate the causal (predictive) influences of multiple latent factors

    C.determine, for a sample of people, how many types the sample represents

    D.identify natural subgroupings among a collection of observations
    The correct answer is B.

    Structural equation modeling is a multivariate technique used to test causal hypotheses about the relationships among a set of factors. Of the answers given, this one best describes the purpose of structural equation modeling. It allows a researcher to identify the underlying (latent) factors that relate to a set of measured variables and the nature of the causal relationships between those factors. Additional information on SEM is provided in the Statistics and Research Design chapter of the written study materials.

    Answer A: Survival analysis is used to evaluate the length of time to a critical event (e.g., relapse, promotion).

    Answer C: Q-technique factor analysis is used to identify types of people in a sample of people.

    Answer D: Cluster analysis is used to identify homogeneous subgroups in a heterogeneous collection of observations.
  206. When conducting a one-way ANOVA to compare the effects of four different diets on weight loss, an F-ratio of _____ suggests that there may be a significant difference between the types of diets?

    A.15.5

    B.0.9

    C.0

    D.-1.5
    The correct answer is A.

    The F-ratio is calculated by dividing the mean square between by the mean square within. Mean square between is a measure of treatment effects plus error, while mean square within is a measure of error only. A treatment effect is suggested when the numerator of the F ratio (mean square between) is larger than the denominator (mean square within) resulting in an F value greater than +1.0.
  207. A multiple regression equation yields a predicted criterion score for an examinee based on the examinee's scores on the predictors included in a test battery. When computing a multiple regression equation, each test is weighted_____.

    A.in direct proportion to its correlation with the criterion and in inverse proportion to its correlation with the other predictors in the test battery

    B.in inverse proportion to its correlation with the criterion and in direct proportion to its correlation with the other predictors in the test battery

    C.in direct proportion to its correlation with the criterion and with the other predictors in the test battery

    D.in inverse proportion to its correlation with the criterion and with the other predictors in the test battery
    The correct answer is A.

    When choosing tests for a test battery used to predict status on a criterion, each test should correlate highly with the criterion but not with the other tests in the battery in order to provide the most useful (nonredundant) information. By computing the multiple regression equation so that each test is weighted in direct proportion to its correlation with the criterion and in inverse proportion to its correlation with other tests, the test with the highest criterion-related validity and the least amount of overlap (correlation) with the other tests will be given the largest weight, while the test with the lowest criterion-related validity and the most overlap with the other tests will be given the smallest weight.

    Answer B: This is the opposite of what you would do to predict a criterion score. When choosing tests for a test battery used to predict status on a criterion, each test should correlate highly with the criterion but not with the other tests in the battery in order to provide the most useful (nonredundant) information.

    Answer C: You would not want other predictors in the test battery to be directly proportionate. When choosing tests for a test battery used to predict status on a criterion, each test should correlate highly with the criterion but not with the other tests in the battery in order to provide the most useful (nonredundant) information.

    Answer D: If each test is weighted in inverse proportion to its correlation with the criterion and the other predictors in the test battery, then each test with the least amount of overlap would have the largest weight and would not provide useful information about the predicted criterion score. When choosing tests for a test battery used to predict status on a criterion, each test should correlate highly with the criterion but not with the other tests in the battery in order to provide the most useful (nonredundant) information.
  208. A researcher would choose an ANCOVA to _____.

    A.statistically remove error variance that is attributable to a known extraneous variable

    B.determine if the effects of an extraneous variable moderate the relationship between the IV and DV

    C.simultaneously assess more than one hypothesis about the IV's effects on the DV

    D.test the effects of an IV on more than one DV without increasing the experiment-wise error rate
    The correct answer is A.

    Researchers use the ANCOVA to statistically remove the effects of an extraneous variable (i.e., error variance) so it is easier to detect the effects of the IV on the DV.

    Answer B: This doesn't describe the specific benefit of using an ANCOVA to remove the effects of an extraneous variable.

    Answer C: This isn't why an ANCOVA is used. Researchers use ANCOVAs to statistically remove the effects of an extraneous variable to make it easier to detect the effects of the IV on the DV.

    Answer D: This describes the MANOVA (multivariate analysis of variance).
  209. Adding a constant to every score in a distribution of scores will_____.

    A.increase the distribution's mean

    B.increase the distribution's standard deviation

    C.increase the distribution's mean and standard deviation

    D.not increase the distribution's mean or standard deviation
    The correct answer is A.

    Adding a constant only affects the mean and other measures of central tendency. It does not affect the variability (spread) of scores in the distribution. As noted in the Statistics and Research Design chapter of the written study materials, adding or subtracting a constant to each score changes the mean and multiplying or dividing by a constant changes both the mean and the standard deviation.

    Answer B: Adding a constant will not affect the variability (spread) of scores in the distribution. It only affects the mean and other measures of central tendency.

    Answer C: Adding a constant would only affect the mean and other measures of central tendency, but would not have an affect on the standard deviation or variability of scores in the distribution.

    Answer D: Adding a constant would affect the mean and other measures of central tendency, but it would not affect the variability (spread) of scores in the distribution or standard deviation.
  210. You would use a factorial ANOVA to statistically analyze data when_____.

    A.the study includes two or more dependent variables

    B.there are more than two levels of a single independent variable

    C.the study includes two or more independent variables

    D.the effects of a confounding variable must be statistically removed
    The correct answer is C.

    You would use a factorial ANOVA to analyze data when a factorial design is used and the dependent variable is measured on an interval or ratio scale. A research design is considered "factorial" when it includes two or more independent variables, and you use a factorial ANOVA to analyze the main effects of each independent variable as well as their interaction(s).

    Answer A: A MANOVA (multivariate analysis of variance) would be the appropriate test when the study includes two or more dependent variables.

    Answer B: A one-way ANOVA would be the appropriate test when there are more than two levels of a single independent variable.

    Answer D: An ANCOVA (analysis of covariance) would be used when the effects of a confounding variable must be statistically removed.
  211. A distribution of scores is _____ when it is more "peaked" than a normal distribution.

    A.platykurtic

    B.leptokurtic

    C.mesokurtic

    D.endokurtic
    The correct answer is B.

    For the exam, you want to know the difference between platykurtic and leptokurtic. A leptokurtic distribution is more "peaked" than a normal distribution (i.e., a larger proportion of scores are in the middle of the distribution).

    Answer A: A platykurtic distribution is flatter than a normal distribution.

    Answer C: A normal distribution is mesokurtic.

    Answer D: Endokurtic is a "made-up" term.
  212. In a normal distribution of test scores, approximately ____% of scores are more than two standard deviations above the mean.

    A.2.5

    B.5

    C.7.5

    D.10
    The correct answer is A.

    In a normal distribution, approximately 95% of scores fall between scores that are -2 and +2 standard deviations from the mean. This means that about 2.5% of scores are less than the score that is 2 standard deviations below the mean and about 2.5% are more than the score that is 2 standard deviations above the mean.

    Answer B: For this question, you must identify the % of scores that fall more than two standard deviations above the mean. Knowing that roughly 95% of scores fall between scores that are -2 and +2 standard deviations from the mean, leaves you with 5% of scores that fall above and below the mean. About 2.5% of scores are less than the score that is 2 standard deviations below the mean and about 2.5% are more than the score that is 2 standard deviations above the mean.

    Answer C: About 95% of scores fall between scores that are -2 and +2 standard deviations from the mean. This means that about 2.5% of scores are less than the score that is 2 standard deviations below the mean and about 2.5% are more than the score that is 2 standard deviations above the mean.

    Answer D: About 95% of scores fall between scores that are -2 and +2 standard deviations from the mean. This means that about 2.5% of scores are less than the score that is 2 standard deviations below the mean and about 2.5% are more than the score that is 2 standard deviations above the mean.
  213. When conducting a one-way ANOVA, you calculate an F-ratio by_____.

    A.dividing the mean square between (MSB) by the mean square within (MSW)

    B.dividing the mean square within (MSW) by the mean square between (MSB)

    C.dividing the mean square total (MST) by the mean square between (MSB)

    D.dividing the mean square total (MST) by the mean square within (MSW)
    The correct answer is A.

    When using the one-way ANOVA to determine if an independent variable has had a significant effect on a dependent variable, you calculate an F-ratio by dividing the mean square between (MSB) by the mean square within (MSW). MSB provides an estimate of variability due to treatment plus error, while MSW provides an estimate of variability due to error only. When the independent variable has had an effect, MSB will be larger than MSW and the F-ratio will be larger than 1.0. The larger the F-ratio, the more likely that the effect of the independent variable is statistically significant.

    Answer B: This is the opposite of how you would calculate an F-ratio. To calculate the F-ratio, you divide the mean square between by the mean square within.

    Answer C: You would not use the MST when calculating an F-ratio. When calculating the F-ratio, you would divide the mean square between by the mean square within.

    Answer D: You would not use the MST when calculating an F-ratio. When calculating the F-ratio, you would divide the mean square between by the mean square within.
  214. Which of the following would be most useful for combining the results of several research studies that each evaluated the effectiveness of the same treatment for a particular disorder?

    A.P-value

    B.Kappa coefficient

    C.Coefficient alpha

    D.Effect size
    The correct answer is D.

    This describes a meta-analysis and an effect size is the index you would calculate when conducting a meta-analysis. It converts data from different studies (which may have different sample sizes, different outcome measures, etc.) to a common metric so you can combine and compare the results of the studies.

    Answer A: Although the p-value indicates the level of significance (probability that the null hypothesis is false), it is not as useful as an effect size for combining the results of different studies.

    Answer B: The kappa coefficient is used to evaluate inter-rater reliability.

    Answer C: Coefficient alpha is used to assess internal consistency reliability
  215. When using the analysis of variance, calculation of the F-ratio involves_____.

    A.comparing group means directly to each other

    B.comparing each group mean to a total mean

    C.comparing true (experimental) variability to error variability

    D.comparing between-group variability to within-group variability
    The correct answer is D.

    The F-ratio is a measure of treatment effects plus measurement error divided by a measure of measurement error only. This is the best description of the F-ratio since the numerator is a measure of between-group variability, while the denominator is a measure of within-group variability.

    Answer A: Unlike the t-test, the ANOVA compares means indirectly rather than directly.

    Answer B: Some of the calculations for the ANOVA do involve comparing group means to a "grand mean," but this is not used to derive the F-ratio itself.

    Answer C: This doesn't describe the F-ratio.
  216. To evaluate the impact of an educational program on each student's mastery of the information presented, which of the following would be most useful?

    A.Norm-referenced scores

    B.Criterion-referenced scores

    C.Standard scores

    D.Ipsative scores
    The correct answer is B.

    To determine if a student has benefited from an educational program, you would most likely want to determine how much of the information presented in the program has been retained and/or to what degree participation in the program has improved the individual's performance on a task. Criterion-referenced scores tell you how well an examinee did in absolute terms (e.g., how many questions he/she answered correctly) and, therefore, would be most useful for the purpose described in the question.

    Answer A: Norm-referenced scores (e.g., standard scores, percentile ranks) tell you how well an examinee is doing compared to other examinees.

    Answer C: Standard scores are a type of norm-referenced score.

    Answer D: Ipsative scores indicate the relative strengths of the different characteristics measured by a test for the individual and would be less useful than criterion-referenced scores for the purpose described in the question.
  217. An item discrimination (D) index of 0 for an item included in an academic achievement test indicates that:

    A.no students answered this item correctly

    B.the item was answered correctly by more low-achieving students than by high-achieving students

    C.the item was answered correctly by more high-achieving students than by low-achieving students

    D.the item was answered correctly by the same number of low- and high-achieving students
    The correct answer is D.

    The item discrimination index (D) indicates the extent to which a test item discriminates between examinees who obtain high versus low scores on the entire test or on an external criterion and ranges in value from +1.0 to -1.0. The item discrimination index is calculated by subtracting the percent of examinees in the low-scoring group who answered the item correctly from the percent of examinees in the high-scoring group who answered the item correctly. D is equal to 0 only when these percents are the same.

    Answer A: When the item discrimination index (D) equals 0, it indicates the item was answered correctly by the same number of low- and high-achieving students. It does not indicate that no students answered the item correctly.

    Answer B: A negative item discrimination index indicates that the item was answered correctly by more low-achieving students than by high-achieving students.

    Answer C: A positive item discrimination index indicates that the item was answered correctly by more high-achieving students than by low-achieving students.
  218. A screening test for a disorder that has a very low base rate in the population is known to have an overall accuracy rate of 98%. When using this test to identify individuals in the general population who have the disorder, it's important to keep in mind that the test will produce:

    A.very few false positives but very many true positives

    B.very few true negatives but very many false negatives

    C.a larger number of false negatives than false positives

    D.a larger number of false positives than false negatives
    The correct answer is D.

    This is a difficult question, but you may have been able to identify the correct answer if you know that a low base rate means that there are very few people in the population who have the disorder, which implies that the most likely predictive error will be to falsely identify those who do not have the disorder as having it. To understand the answer, it is best to use a concrete example. Assume that the base rate for the disorder is 1% and that you test a random sample of 10,000 people with the screening test. In this situation, 100 people will have the disease, and the test (which has a 98% accuracy rate) will correctly identify 98 of them - i.e., 98 will be true positives and 2 will be false negatives. When the base rate for the condition is 1%, 9,900 of the 10,000 people tested will not have the disease, and the test (with a 98% accuracy rate) will correctly identify 9,702 of them - i.e., 9,702 will be true negatives and the remaining 198 will be false positives. In other words, there will be more false positives than false negatives - and this will be true whenever the predictor has a high accuracy rate and the base rate is less than 50%.

    Answers A, B, and C: To identify the correct answer you needed to know that a low base rate means that there are very few people in the population who have the disorder, which implies that the most likely predictive error will be to falsely identify those who do not have the disorder as having it (a larger number of false positives than false negatives).
  219. In a factor matrix, the factor loading for Test A and Factor II is .70. This means that:

    A.70% of variability in Test A is accounted for by Factor II

    B.49% of variability in Test A is accounted for by Factor II

    C.only 70% of variability in Test A is accounted for by the factor analysis

    D.only 49% of variability in Test A is accounted for by the factor analysis
    The correct answer is B.

    A factor loading is the correlation coefficient for a test and an identified factor. A factor loading can be interpreted by squaring it to obtain a measure of shared variability. When the factor loading is .70, this means that 49% (.70 squared) of variability in the test is accounted for by the factor.

    Answers A, C, and D: When the factor loading is .70, this means that 49% (.70 squared) of variability in the test is accounted for by the factor.
  220. When using the multitrait-multimethod matrix to evaluate the construct validity of a newly developed test, a __________ coefficient provides evidence of the test's divergent (discriminant) validity.

    A.large monotrait-heteromethod

    B.large heterotrait-monomethod

    C.small monotrait-heteromethod

    D.small heterotrait-monomethod
    The correct answer is D.

    The answers to this question list two of the four types of correlation coefficients included in a multitrait-multimethod matrix. A small heterotrait-monomethod coefficient provides evidence of a test's divergent validity - i.e., it shows that the test does not correlate with a measure of another trait that it should not correlate with.

    Answer A: The monotrait-heteromethod coefficient indicates the correlation of two different methods that measure the same trait. A large monotrait-heteromethod coefficient provides evidence of a test's convergent validity. For example, if the test being validated is a self-rating scale of self-esteem and it has a high correlation with a previously validated parent rating scale of self-esteem, this would suggest that the test being validated is actually measuring self-esteem.

    Answer B: The heterotrait-monomethod coefficient indicates the correlation between two similar methods that measure different traits. A large heterotrait-monomethod coefficient indicates that a test has a lack of divergent validity. For example, if the test being validated is a self-rating scale of self-esteem and it has a high correlation with a self-rating scale of neuroticism, this would suggest that the measure of self-esteem may be measuring something other than self-esteem since these two traits should not correlate.

    Answer C: A small monotrait-heteromethod coefficient would suggest that the test lacks convergent validity.
  221. Assuming that the following scores are all from the same normal distribution of scores, which of the following lists the scores in order from lowest to highest?

    A.Z-score of +1.0, percentile rank of 70, T-score of 80

    B.Z-score of +.75, percentile rank of 84, T-score of 65

    C.Z-score of +1.25, percentile rank of 95, T-score of 55

    D.Z-score of +.50, percentile rank of 98, T-score of 60
    The correct answer is B.

    For the exam, you want to be familiar with the relationship between z-scores, percentile ranks, and T-scores. Converting the scores to standard deviation units would have helped you identify the correct answer to this question. A z-score of +.75 is 3/4ths of a standard deviation above the mean; a percentile rank of 84 is one standard deviation above the mean; and a T-score of 65 is 1-1/2 standard deviations above the mean. Therefore, this answer lists the scores in order from lowest to highest.

    Answer A: A z-score of +1.0 is one standard deviation above the mean, a percentile rank of 70 is below one standard deviation above the mean, and a T-score of 80 is three standard deviations above the mean.

    Answer C: A z-score of +1.25 is 1-1/4 standard deviations above the mean, a percentile rank of 95 is more than one standard deviation above the mean, and a T score of 55 is one-half standard deviation above the mean.

    Answer D: A z-score of +.50 is one-half standard deviation above the mean, a percentile rank of 98 is two standard deviations above the mean, and a T-score of 60 is one standard deviation above the mean.
  222. When item response theory has been used as the basis for test construction, an examinee's score on the test provides information about his/her:

    A.future status on an external criterion

    B.status on a latent trait or ability

    C.performance relative to other examinees

    D.performance relative to a prespecified standard
    The correct answer is B.

    Item response theory differs from classical test theory in several ways, including the interpretation of an examinee's scores. One of the primary characteristics of item response theory is that it is based on the assumption that "the performance of an examinee on a test item can be explained (or predicted) by a set of factors called 'traits,' 'latent traits,' or 'abilities.'" Source: D. H. Henard, Item response theory, in L. G. Grimm and P. R. Yarnold (eds.), Reading and understanding more multivariate statistics, Washington, DC, APA, 2000.

    Answer A: Predictive validity is a type of criterion-related validity used when the purpose of testing is to predict future status on a criterion.

    Answer C: Norm-referenced interpretation involves comparing an examinee's performance to other examinees.

    Answer D: Criterion-referenced interpretation provides information about an examinee's score in relation to a prespecified standard.
  223. All other things being equal, which of the following types of tests would be expected to have the lowest reliability?

    A.7-response multiple choice

    B.3-response multiple choice

    C.True-false

    D.Free recall
    The correct answer is C.

    A test's reliability is an index of the degree to which scores on the test are free from random error (chance factors) and indicative of examinees' true scores. The greater the probability that an examinee can answer an item correctly by guessing (i.e., by chance), the lower a test's reliability. On a true-false test, the probability that the correct answer can be selected by chance is 50% (1/2). In other words, there is a higher probability that an examinee can guess a correct answer on a true-false test than on the other tests, and thus, an examinee's score on a true-false test will reflect, to a greater degree, error rather than his/her true score.

    Answer A: The probability that an examinee can guess a correct answer on a 7-response multiple choice test is 14% (1 in 7).

    Answer B: The probability that an examinee can guess a correct answer on a 3-response multiple choice exam is 33% (1 in 3).

    Answer D: The probability that an examinee can guess a correct answer on a free recall test is close to zero.
  224. To construct the 68% confidence interval for an examinee's obtained test score, you would need the examinee's score and:

    A.the test's mean

    B.the standard deviation

    C.the standard error of measurement

    D.the standard error of estimate
    The correct answer is C.

    To construct a confidence interval around an obtained test score, you need the standard error of measurement (which is calculated from the test's standard deviation and reliability coefficient). To construct a 68% confidence interval, you add and subtract one standard error of measurement to and from the examinee's obtained test score.

    Answer A: The mean is not needed to construct a confidence interval.

    Answer B: The standard deviation is one of the elements needed to calculate the standard error of measurement but, by itself, cannot be used to construct a confidence interval.

    Answer D: The standard error of an estimate is used to construct a confidence interval around a predicted criterion score.
  225. A normal distribution of raw scores has a mean of 106 and a standard deviation of 10. In this distribution, which of the following scores is equivalent to a raw score of 126? 

    A.a percentile rank of 98

    B.a T-score of 55

    C.a z-score of +2.5

    D.a stanine score of 6
    The correct answer is A.

    In the distribution described in this question, a raw score of 126 is two standard deviations above the mean. In a normal distribution, a percentile rank of 98 is about two standard deviations above the mean.

    Answer B: A T-score of 55 is one-half standard deviation above the mean.

    Answer C: Z-scores are interpreted directly in terms of standard deviation units, which means that a z-score of +2.5 is 2-1/2 standard deviations above the mean.

    Answer D: A stanine score of 6 is between the mean and the score that is one standard deviation above the mean.
Author
mdawg
ID
361843
Card Set
EPPP - Practice Exam 2
Description
Updated